Question 1 of 67 A young patient is tilting her pelvis while walking. They are having difficulty maintaining independent mobility. Which clinical test is used to assess the hip abductors?

Barlow’s test Ober’s test Ortolani’s test Thomas’ test Trendelenburg’s test

Explanation

Trendelenburg’s test Trendelenburg’s test assesses for weak or paralysed the hip abductor muscles. This test is performed with the patient standing. The patient is asked to raise one leg; the test is positive if the pelvis on the raised side drops. A positive test suggests weakness of the abductors of the other hip.

Barlow’s test Barlow’s test is performed on infants to screen for developmental dysplasia of the hip and assessing the stability of the hip.

Ober’s test Ober’s test is performed to assess the tightness of the iliotibial band.

Ortolani’s test Ortolani’s test is performed on infants to screen for developmental dysplasia of the hip, which is performed along with the Barlow’s test.

Thomas’ test

Thomas’ test is performed to assess for hip flexion contracture

Question 2 of 67

A 68-year-old man presents with mechanical hip pain. He undergoes routine plain AP and lateral radiography of the hip. What is the first radiographic sign to appear in osteoarthritis?

Narrowing joint space

Osteopenia

Osteophytes

Soft tissue swelling

Subchondral sclerosis

Explanation

Narrowing joint space Osteoarthritis is a degenerative joint disorder, which results from a loss of hyaline cartilage in synovial joints. The first radiographic sign of osteoarthritis is non-uniform joint space narrowing.

Osteopenia Osteopenia is a condition in which bone mineral density is lower than normal.

Osteophytes Osteophyte formation is a radiographic sign of osteoarthritis, but it is not the first to appear. As the disease progresses, subluxation may occur and osteophytes may form.

Soft tissue swelling Rest pain, stiffness and bony swelling are clinical features of osteoarthritis. Soft tissue swelling is more commonly associated with rheumatoid arthritis.

Subchondral sclerosis Subchrondal sclerosis occurs as cartilage loss increases and appears as an area of increased density on the radiograph. The classic radiographic signs of osteoarthritis are joint space narrowing, subchondral sclerosis, osteophytes and subchondral cysts. Osteopenia and soft tissue swelling are often associated with rheumatoid arthritis.

Question 3 of 67

A 10-year-old girl is treated for a tibia and fibula fracture with above-knee plaster. She is recovering on the ward, but the nurses are concerned and have asked you to review her overnight. What early clinical signs of compartment syndrome in the leg do you look for?

Loss of the dorsalis pedis pulse

Mottling of the overlying skin

Pain out of proportion to injury on passive dorsiflexion of the foot

Paralysis of the foot

Reduced sensation in the foot

Explanation

Pain out of proportion to injury on passive dorsiflexion of the foot Compartment syndrome is a clinical diagnosis defined as increased pressure within a fascial compartment that causes reduced tissue perfusion and cellular damage. Dorsiflexion of the foot stretches muscles in the posterior compartment, which causes an out-of-proportion pain response in the presence of high compartment pressures as myoneural blood supply is compromised. Clinical signs of compartment syndrome include pain out of proportion of the injury, pain on passive starching, pain despite analgesia, pain despite elevation and pain despite removal of constrictive dressings.

Loss of the dorsalis pedis pulse Pulses are normally present in compartment syndrome. Absent pulses are usually due to systemic hypotension, arterial occlusion or vascular injury. In the context of compartment syndrome absent pulses are a late finding.

Mottling of the overlying skin Mottling of the skin would be a late clinical sign of compartment syndrome. Local trauma causes bleeding and soft tissue swelling, which compromises venous return and gives rise to a mottled appearance of skin.

Paralysis of the foot The loss of motor function is a late sign of compartment syndrome. Local trauma causes bleeding and soft tissue swelling, which increases compartmental pressures. This compromises venous return and propagates swelling and further increases compartmental pressures causing microvascular occlusion resulting in myoneural ischaemia.

Reduced sensation in the foot The sensory loss is a late sign of compartment syndrome. Local trauma causes bleeding and soft tissue swelling, which increases compartmental pressures. This compromises venous return and propagates swelling and further increases compartmental pressures causing microvascular occlusion resulting in myoneural ischaemia.

Question 4 of 67 A 30-year-old man complains of having to swing his body to help him walk after a long session of exercise in the gym. Trendelenburg’s test is performed. If the left side of the patient’s pelvis drops while he stands on his right leg, what is the likely cause?

A shortened right femur

Weakness of the right hip abductor muscles

Weakness of the left hip abductor muscles

An injury to the left gluteus medius muscle

bilateral

Explanation

Weakness of the right hip abductor muscles Trendelenburg’s test assesses for weak or paralysed hip abductor muscles. This test is performed with the patient standing. The patient is asked to raise one leg; the test is positive if the pelvis on the raised side drops. A positive test suggests weakness of the abductors of the other hip. When the gluteal muscles are compromised (weakened as a result of a hip dislocation, due to paralysis, or following hip operations) with functional deficits, they are no longer able to support the pelvis on the weight-bearing side. The pelvis than drops down on the normal, non- weight-bearing side.

A shortened right femur In the case of a shortened femur the upper body is shifted slightly over the leg in the stance phase. If a patient has a shortened right femur and was standing on their right leg, their body weight would be shifted to right raising the pelvis on the opposite side.

Weakness of the left hip abductor muscles Trendelenburg’s test assesses the hip abductor muscles on the side that the patient is standing, therefore weakness of the left hip abductor muscles would result in a patient’s pelvis dropping while standing on their left leg.

An injury to the left gluteus medius muscle Trendelenburg’s test assesses the hip abductor muscles, these include the gluteus medius, tensor fasciae latae, gluteus minimus, piriformis and obturator internus. Injury to the left gluteus medius would result in the right side of the pelvis dropping when standing on the left leg. bilateral hip dysplasia Patients with bilateral hip dysplasia walk with a ‘waddling gait’ (myopathic gait) and the Trendelenburg’s test will be positive on both sides. Trendelenburg’s test is used to assess stability of the hip. An abnormal Trendelenburg’s test is seen with any painful disorder of the hip. It is also present with a dislocated or subluxed hip and in other conditions in which the proximal femoral anatomy is abnormal (short femoral neck with high riding trochanter). Weak abductor muscles will also lead to an abnormal test. The abductors of the hip include the piriformis, gluteus medius and minimis muscles. The pectineus muscle adducts the hip. The other adductors of the hip are the gracilis, and adductor longus, brevis and magnus muscles.

Question 5 of 67

A 38-year-old builder’s labourer sustained a severe fracture of his left , which damaged the ulnar nerve behind the medial epicondyle of the humerus. A month later, he still has total ulnar nerve paralysis. Which clinical sign is most likely to be present on examination?

Sensory loss over the ulnar 3½ digits Inability to grip a sheet of paper between his fingers Excessive sweating over the ulnar border of the left hand Ring and little fingers on the affected side are held in the claw position Marked wasting of the thenar eminence

Explanation

Inability to grip a sheet of paper between his fingers To grip a sheet of paper requires finger adduction and the functioning of the palmar interossei muscles, which are supplied by the deep branch of the ulnar nerve.

Sensory loss over the ulnar 3½ digits Damage to the ulnar nerve would result in sensory loss over the ulna 1½ digits.

Excessive sweating over the ulnar border of the left hand Injury to the ulnar nerve may result in sympathetic interruption, with absence of sweating in the affected area.

Ring and little fingers on the affected side are held in the claw position Although the ring and little finger are held in the clawed position when the ulnar nerve is injured at the wrist, a high injury at the level of the elbow paralyses the long flexors to these two fingers and results in the loss of this sign (the ulnar paradox).

Marked wasting of the thenar eminence

The recurrent motor branch of the median nerve supplies the thenar eminence. Wasting of the thenar eminence would suggest damaged to the median nerve, and can be seen in carpal tunnel syndrome.

Question 6 of 67 A 49-year-old woman has been complaining of a tingling feeling in her right hand at night and is under the impression that her hand is swollen although there is no obvious oedema. In the last few days she has noticed numbness in her right index finger and the tip of her thumb, especially while working. Which nerve is most likely to be responsible for her symptoms?

Ulnar nerve Median nerve Radial nerve Nerve root C7 Nerve root C8

Explanation

Median nerve The median nerve supplies sensation to the palm and the palmer surface of the radial 3½ digits. The symptoms described are suggestive of carpal tunnel syndrome. This often spares sensory loss to the palm as the palmer branch of the median nerve comes off before it enters the carpal tunnel. The digital branches and the thenar muscles will be affected. The usual presentation of carpal tunnel syndrome is with acroparaesthesias. This consists of numbness, tingling and burning sensations felt in the hand and fingers; the pain sometimes radiates up the forearm as far as the elbow or even as high as the shoulder or root of the neck. Although the paraesthesias are sometimes restricted to the radial fingers, they may affect all the digits as some fibres from the median nerve are distributed to the fifth finger through a communication with the ulnar nerve in the palm. The attacks of pain and paraesthesias are most common at night and often wake the patient from sleep. They are then relieved by shaking the hand. The hand tends to feel numb and useless on waking in the morning but recovers after it has been used for some minutes. The symptoms may recur during the day following use, or at other times if the patient sits with the hands immobile. Such symptoms of acroparaesthesias may persist for many years without the appearance of symptoms of median nerve damage. In other patients, weakness of the thenar muscles develops, particularly with abduction of the thumb, and is associated with atrophy of the lateral aspect of the thenar eminence. Sensory loss may appear over the tips of the median innervated fingers.

Ulnar nerve The ulnar nerve supplies sensation to the ulna 1½ digits, motor function to the intrinsic muscles of the hand, the ulna half of flexor digitorum profundus and the flexor carpi ulnaris muscle in the forearm.

Radial nerve The radial nerve supplies sensation to the first dorsal web space in the hand and motor function to the extensor muscle of the forearm.

Nerve root C7 The C7 nerve root is most strongly associated with elbow extension and sensation along the middle finger.

Nerve root C8

The C8 nerve root is most strongly associated with finger flexion and sensation along the ring and little finger. Question 7 of 67 An 18-year-old falls out of a tree and injures his arm. Subsequently his hand appears clawed. On examination, his hand is extended at all the metacarpal (MCP) joints and all the interphalangeal joints are in fixed flexion. The wrist flexors appear to be slightly weaker. What is likely to be the injury?

Ulnar nerve injury C8, T1 root injury Median nerve injury C5, C6 root injury Radial nerve injury

Explanation

C8, T1 root injury Lower brachial plexus injuries (C8, T1) are caused by traction on an abducted arm resulting in Klumpke’s palsy. The classical presentation is of a claw hand in which the forearm is supinated and the wrist and all fingers are flexed. This is caused by the paralysis of the intrinsic muscles of the hand and the flexors of the wrist and fingers. Involvement of T1 may also result in Horner syndrome.

Ulnar nerve injury The ulnar nerve originates from C8–T1 and innervates the flexor carpi ulnaris muscle and the flexor digitorum profundus muscle of the ring and little finger, as well as the intrinsic muscles of the hand. Ulnar nerve injury results in clawing of the ring and little finger from denervation of the ulna two lumbricals and unopposed action of the flexor digitorum profundus resulting in extended metacarpophalangeal joints and flexed interphalangeal joints. If the injury occurs more proximally, the flexor digitorum profundus to the ring and little finger is also denervated and the claw-like appearance is less apparent, this is known as the ulnar paradox.

Median nerve injury The median nerve originates from C5–T1 segments and innervates the flexors of the forearm (except flexor carpi ulnaris and flexor digitorum profundus muscles to the ring and little finger) and the lumbricals 1 and 2, opponens pollicis, abductor pollicis brevis and flexor pollicis brevis muscles. Distal median nerve injuries can result in an ape hand deformity at rest, which presents as an inability to abduct the thumb, leaving it resting in adduction. Proximal median nerve injuries result in the hand of benediction when attempting to form a fist. The index and middle finger remain extended at the metacarpophalangeal joints and interphalangeal joints because of denervation of the lateral lumbricals and flexor digitorum profundus respectively.

C5, C6 root injury Upper brachial plexus injuries (C5, C6) are caused by excessive lateral flexion of the neck resulting in Erb’s palsy. The classical presentation is of a waiter’s tip position whereby the limb hangs by the side, medially rotated with the forearm extended and supinated.

Radial nerve injury

The radial nerve originates from C5–T1 segments and innervates the extensors of the arm and forearm. Radial nerve injury results in a wrist drop due to the inability to extend the hand and fingers.

Question 8 of 67 A 37-year-old woman is seen in the hand clinic with paraesthesia in the thumb, index and middle fingers together with a positive Tinel’s and Phalen’s test at the wrist. A diagnosis of carpal tunnel syndrome is made. Which one of the following is correct with regard to this diagnosis?

May be caused by metabolic disorders such as gout, diabetes and hypothyroidism May be caused by inflammation of palmaris longus tendon May cause anaesthesia of the little finger May cause wasting of abductor digiti minimi (ADM) Occludes the ulnar artery

Explanation

May be caused by metabolic disorders such as gout, diabetes and hypothyroidism Carpal tunnel syndrome presents with paraesthesia of the radial 3.5 digits and weakness of the thenar muscle from compression of the median nerve within the carpal tunnel. Sensation within the palm is unaffected, as the palmer branch of the median nerve does not travel through the carpal tunnel. Tinel’s, Phalen’s and Durkan’s tests are all provocation tests used clinically to assess for carpal tunnel syndrome. Carpal tunnel syndrome is associated with pregnancy, arthritis, acromegaly, hypothyroidism, diabetes, gout, alcoholism, tumours and connective tissue disorders.

May be caused by inflammation of palmaris longus tendon

The palmaris longus tendon runs superficially within the forearm and inserts onto the transverse carpal ligament, which forms the roof of the carpal tunnel. It can be used clinically for tendon grafting, although about 10% of the general population lack a palmaris longus tendon. The median nerve in the forearm runs between the flexor digitorum superficialis and flexor digitorum profundus, deep to the palmaris longus tendon.

May cause anaesthesia of the little finger

The ulnar nerve supplies sensation to the little finger and ulna half of the ring finger, and passes through Guyon’s cannel at the wrist. Guyon’s canal is ulnar to the carpal tunnel and contains the ulnar artery and nerve. The canal is bordered on the ulnar side by the pisiform and on the radial side by the hook of the hamate.

May cause wasting of abductor digiti minimi (ADM) The abductor digiti minimi is a muscle of the Hypothenar eminence in the hand and is supplied by the ulnar nerve, which is unaffected in carpal tunnel syndrome. The median nerve enters the hand via the carpal tunnel and dives into two branches; the recurrent branch (innervating thenar muscles) and the palmar digital branch (innervating palmar surface and lateral three and a half digits). Wasting of the thenar muscles, in particular abductor pollicis brevis muscles can be seen in carpal tunnel syndrome, but this would not typically give rise to wasting of abductor digiti minimi muscle.

Occludes the ulnar artery

Ulnar artery occlusion (also known as hypothenar hammer syndrome) is a rare post-traumatic digital ischaemia due to thrombosis of the ulnar artery at Guyon’s canal. It presents with pain over the hypothenar eminence and paraesthesia in an ulnar nerve distribution in the hand.

Question 9 of 67

A 54-year-old man is seen in the Out-patient Hand Clinic with a long history of progressively worsening flexion contracture of his left ring and little finger. He cannot grip objects and has difficulty doing his buttons. Which one of the following is correct?

Central cords displace the neurovascular bundle Is caused by contraction of the palmar fascia Is more common in people of African origin than caucasians May be associated with retroperitoneal fibrosis Most commonly affects the little finger

Explanation

Is caused by contraction of the palmar fascia

Dupuytren’s contracture is a common benign fibroproliferative disorder affecting the palmer fascia. It is a progressive disorder, presenting initially with thickening of the palmer fascia, after which a firm painless nodule develops, which normally precedes cord formation, which causes gradual contraction.

Central cords displace the neurovascular bundle

In Dupuytren’s contracture, central cords are found in the digits and insert onto the flexor sheath at the proximal interphalangeal joint (PIPJ) level causing metacarpophalangeal joint (MCPJ) contracture. Central cords are not involved with the neurovascular bundle. Spiral cords travel up the sides of the digits displacing the neurovascular bundle centrally and superficially, and cause contractures at the PIPJ.

Is more common in people of African origin than caucasians

Dupuytren’s contracture commonly affects elderly men of northern European descent with a pronounced genetic predisposition. It is rare in those people from African origin.

May be associated with retroperitoneal fibrosis

Dupuytren’s contracture is associated with knuckle pads (Garrod’s pads), penile fibrous plaques (Peyronie’s disease) and plantar fibromatosis (Ledderhose’s disease). Retroperitoneal fibrosis, however, is not associated with Dupuytren’s disease.

Most commonly affects the little finger

The ring finger is most commonly affected in Dupuytren’s contracture followed by the little finger, middle finger, then index finger.

Question 10 of 67 A 32-year-old man is admitted to the day case unit for release of carpal tunnel under local anaesthesia. Your consultant has asked that you start the procedure and will be in shortly. Which one of the following is correct?

Carpal tunnel syndrome is associated with Bennett’s fracture

Carpal tunnel syndrome is associated with Golfer’s elbow

During the procedure, a longitudinal incision is made ulnar to the interthenar eminence and should not cross the distal wrist crease.

Opening of Guyon’s canal leads to release of pressure on the median nerve.

Palmaris longus passes through the carpal tunnel. Explanation

During the procedure, a longitudinal incision is made ulnar to the interthenar eminence and should not cross the distal wrist crease.

The skin incision for a carpal tunnel decompression is made in line with the radial border of the ring finger from the distal wrist crease to Kaplin’s line (a line from the apex of the interdigital fold between thumb and index finger to the hook of the hamate). Crossing the wrist crease at right angles may result in a hyper-trophic and painful scar, as well as a wrist flexion contracture.

Carpal tunnel syndrome is associated with Bennett’s fracture A Bennett’s fracture is an intra-articular fracture of the base of the first metacarpal, which extends into the carpometacarpal joint and is not associated with carpal tunnel syndrome

Carpal tunnel syndrome is associated with Golfer’s elbow Golfer’s elbow is also known as medial epicondylitis commonly caused by repetitive activities that cause micro- trauma to the insertion of the flexor-pronator mass. Golfer’s elbow is not associated with carpal tunnel syndrome.

Opening of Guyon’s canal leads to release of pressure on the median nerve. Opening the Guyon’s canal leads to release of pressure on the ulnar nerve. Guyon’s canal is medial to the carpal tunnel and contains the ulnar artery and ulnar nerve. The canal is bordered on the ulnar side by the pisiform and on the radial side by the hook of the hamate.

Palmaris longus passes through the carpal tunnel. The palmaris longus tendon runs superficially within the forearm and inserts onto the transverse carpal ligament, and does not pass through the carpal tunnel. The deep carpal arch forms the floor of the carpal tunnel and the transverse carpal ligament forms the roof. The scaphoid and trapezium and the hook of the hamate form the lateral border and the pisiform the medial border. The carpal tunnel contains the tendon of the flexor pollicis longus, four tendons of the flexor digitorum profundus, four tendons of the flexor digitorum superficialis and the median nerve.

Question 11 of 67 The stabilises the otherwise highly unstable shoulder joint. There are four of these mucles in total: infraspinatus, supraspinatus, teres minor and subscapularies. Which muscle of the rotator cuff is mainly involved in impingement of the shoulder?

Infraspinatus Supraspinatus Teres minor Subscapularis None of the above

Explanation

Supraspinatus

The supraspinatus muscle has its origin at the supraspinous fossa and its tendon passes through the subacromial space before attaching to the superior facet on the greater tuberosity. The suprascapular nerve from the upper trunk of the brachial plexus innervates the supraspinatus muscle and its action initiates shoulder abduction. Shoulder impingement results from extrinsic compression of the supraspinatus tendon as it passes through the subacromial space resulting in a ‘painful arc’ occurring during 50° to 130° of abduction.

Infraspinatus

The infraspinatus muscle has its origin at the infraspinous fossa and inserts onto the middle facet of the greater tuberosity. The suprascapular nerve from the upper trunk of the brachial plexus innervates the infraspinatus muscle and its action is the external rotation of the shoulder. It is not involved with shoulder impingement.

Teres minor

The teres minor muscle has its origin at the lateral border of the scapular and inserts onto the inferior facet of the greater tuberosity. The axillary nerve supplies the teres minor muscle and its action is the external rotation of the shoulder. The teres minor muscle is not involved with shoulder impingement.

Subscapularis

The subscapularis muscle has it origin from the subscapular fossa and inserts on to the lesser tuberosity. The upper and lower subscapularis nerves from the posterior cord of the brachial plexus supply the subscapularis muscles and its action is to internally rotate the shoulder.

None of the above

The shoulder joint is a synovial ball and socket joint surrounded by a think and loose fibrous capsule that permits a wide range of movement. The blood supply of the shoulder joint is derived from branches of the anterior and posterior circumflex humeral arteries from the axillary artery and the suprascapular artery from the subclavian artery. The rotator cuff consists of the supraspinatus, infraspinatus, teres minor, and subscapularis. Shoulder impingement results from supraspinatus tendonitis, which normally presents with a ‘painful arc’ occurring during 50° to 130° of abduction.

Question 12 of 67 A poorly fitting crutch has injured the posterior cord of the brachial plexus. The patient attends the fracture clinic with new onset of neurological deficit in the affected upper limb. Which one of the following clinical pictures is this likely to produce?

Weak shoulder abduction and wrist drop Lack of sensation of half the ring finger and the whole little finger together with thenar muscle wasting The upper limb adopting the 'waiters tip' position Winging of the scapula Weak elbow flexion and the 'Benetictine sign'

Explanation

Weak shoulder abduction and wrist drop

The posterior cord of the brachial plexus lies posterior to the axillary artery and has five nerves arising from it. One of the five nerves is the axillary nerve, which innervates the deltoid muscle. Damage to this nerve will cause weakness in shoulder abduction. Another nerve arising from the posterior cord is the radial nerve, which innervates the extensors of the arm and forearm. Damage to the radial nerve will present with a wrist drop.

Lack of sensation of half the ring finger and the whole little finger together with thenar muscle wasting

The median nerve arises from the lateral and medial cords of the brachial plexus. Injury to the medial cord of the brachial plexus can result from a subcoracoid dislocation of the humerus. The medial cord of the brachial plexus gives rise to the medial root of the median nerve and the ulnar nerve damage to which would produce sensory loss to the ulnar one and half digits and wasting of the thenar muscles.

The upper limb adopting the 'waiters tip' position The clinical presentation of a ‘waiter’s tip’ position in which the limb hangs by the side, medially rotated with the forearm extended and pronated, results from Erb’s palsy. Erb’s palsy is the result of an upper brachial plexus injury, which can be causes by traumatic excessive lateral flexion of the neck.

Winging of the scapula

Winging of the scapula is a result of a long thoracic nerve injury and paralysis of the serratus anterior muscle.

Weak elbow flexion and the 'Benetictine sign'

Injury to the lateral cord of the brachial plexus can result from dislocation of the humerus. The lateral cord of the brachial plexus gives rise to the musculocutaneous nerve and the lateral root of the median nerve. The musculocutaneous nerve supplies the bicep brachii, brachialis and coracobrachialis muscles, which are the main elbow flexors. The lateral root of the median nerve supplies the forearm flexors, denervation of radial two flexor digitorum profundus muscles results in the hand of benediction when attempting to make a fist as the index and middle finger remain extended at proximal interphalangeal joint and distal interphalangeal joint.

Question 13 of 67

A man presents to your orthopaedic clinic with a ‘claw hand’ following an injury to his forearm. He is a manual worker and has found that the defect is life changing, he wants a diagnosis. Which nerve roots or their combinations are associated with this neurological injury?

C6, C7 and C8 C7, C8 and T1 C6 and C7 C7 and C8 C8 and T1

Explanation C8 and T1 The claw hand of a distal ulnar nerve palsy is caused by paralysis of the interossei and the ulna two lumbricals with the resultant unopposed action of extensors and flexor digitorum profundus (which would also be affected in a proximal ulnar nerve injury, eg at the elbow). The ulnar nerve is the direct continuation of the medial cord (C8 and T1).

C6, C7 and C8 C6 gives rise to musculocutaneous nerve and the lateral root of median nerve. C7 gives rise to the axillary and radial nerve. C8 gives rise to the ulna and medial root of median nerve.

C7, C8 and T1 C7 gives rise to the axillary and radial nerve. C8 and T1 gives rise to the ulnar and medial root of median nerve.

C6 and C7 C6 gives rise to musculocutaneous nerve and the lateral root of median nerve. C7 gives rise to the axillary and radial nerve.

C7 and C8 C7 gives rise to the axillary and radial nerve. C8 gives rise to the ulna and medial root of median nerve.

Question 14 of 67

A 17-year-old woman is seen in an orthopaedic clinic after being referred by the GP with problems using her left hand. Of note she reports a snowboarding accident in the Alps approximately 1 month ago for which she was diagnosed with a dislocated left elbow. This was manipulated in a local hospital, but she has had ongoing problems with the left hand. On examination she has good wrist extension, normal thumb abduction and is able to flex the elbow. However, it is noted Froment’s test is positive. Which nerve is Froment’s test assessing?

Axillary nerve Median nerve Musculocutaenous nerve Radial nerve Ulnar nerve

Explanation

Ulnar nerve Froment’s test assesses the motor function of the ulnar nerve by testing the action of adductor pollicis. The patient is asked to hold a piece of paper between the thumb and a flat palm as the paper is pulled away. An abnormal test is demonstrated by recruitment of the flexor pollicis longus muscle, seen with flexing at the interphalangeal joint of the thumb to maintain grip pressure. Assessing sensation of the lateral border of the little finger can test the sensory function of the ulnar nerve. The ulnar nerve runs posterior to the medial epicondyle within the cubital tunnel at the elbow and is prone to injury following elbow dislocation.

Axillary nerve The axillary nerve can be assessed by testing the deltoid function (shoulder abduction) and sensation in the regimental badge area (the skin covering the inferior region of the deltoid), which is innervated by the superior lateral cutaneous nerve, a branch of the axillary nerve. The axillary nerve can be damaged following a shoulder dislocation.

Median nerve Getting the patient to make an ‘OK’ sign ensuring the tip of the index finger and thumb touch assesses median nerve. This tests the function of the index finger flexor digitorum profundus (FDP) and flexor pollicis longus (FPL) muscles, which are supplied by the anterior interosseous nerve, a branch of the median nerve. Assessing sensation of the radial border of the index finger can test the sensory function of the median nerve. The median nerve can be damaged following a displaced distal radius fracture, both bone forearm fracture and or supracondylar fracture.

Musculocutaenous nerve Testing the bicep brachii, brachialis and coracobrachialis muscles with elbow flexion and forearm supination assesses the motor function of the musculocutaneous nerve. Assessing sensation of the lateral side of the forearm from the elbow to wrist tests the sensory function of the lateral cutaneous nerve of the forearm, a branch of the musculocutaneous nerve.

Radial nerve Getting the patient to extend their wrist or to give a ‘thumbs up’ assesses the function of the wrist extensors and extensor pollicis longus (EPL) respectively, which are supplied by the posterior interosseous nerve, a branch of the radial nerve. Assessing sensation of the first dorsal web space can test the sensory function of the superficial branch of the radial nerve. The radial nerve can be injured following a mid-shaft humerus fracture, as it passes through the radial groove.

Question 15 of 67

A 43-year-old woman presents with pain and paraesthesia of the medial aspect of the foot, exacerbated by walking. She is diagnosed with tarsal syndrome and receives appropriate management. Which nerve has been injured?

Common fibular nerve

Femoral nerve Lateral cutaneous nerve of the thigh

Sural nerve

Tibial nerve

Explanation

Tibial nerve The tibial nerve originates from the sciatic nerve. It runs inferior to the flexor retinaculum of the foot alongside the tibial artery and vein, the tibialis posterior muscle, and the flexor hallucis longus and flexor digitorum longus muscles. If the nerve is compressed within the tarsal tunnel, then the clinical syndrome described above can result.

Common fibular nerve The common fibular (peroneal) nerve branches into the deep and superficial fibular nerves supplying the anterior and lateral compartments of the lower limb respectively. Motor function can be assessed by ankle dorsal flexion, which tests the tibialis anterior muscle. Sensory function can be assessed by testing sensation to the dorsum of the foot, the deep fibular nerve innervates the first dorsal web space and the rest is supplied by the superficial fibular nerve. The common peroneal nerve can be injured by a fibular neck fracture, an incorrectly fitted below knee plaster or external compression from being in a lithotomy boot during general anaesthesia.

Femoral nerve The femoral nerve supplies the muscles of anterior compartment of the thigh, which assist in hip flexion and knee extension. The femoral nerve also supplies sensation to the skin overlying the anterior thigh. Clinically the femoral nerve and its branches can be blocked to interrupt pain signals, particularly useful in hip fracture patients.

Lateral cutaneous nerve of the thigh The lateral cutaneous nerve of the thigh is a nerve from the lumbar plexus (L2–3) that innervates the skin on the lateral part of the thigh.

Sural nerve The sural nerve originates from branches from the tibial and common peroneal nerve, it travels along with the short saphenous vein and supplies sensation to the lateral aspect of the foot and ankle.

Question 16 of 67 A 1-day-old boy is referred to the paediatricians with omphalocoele, microcephaly, difficulty feeding and cryptorchidism. He is diagnosed with trisomy 18 after further investigation. Which lower limb abnormality is associated with this condition?

Amelia

Congenital talipes equinovarus

Genu valgus

Genu varus

Meromelia

Explanation

Congenital talipes equinovarus Trisomy 18 is also known as Edwards’ syndrome, and presents with a variety of developmental defects. It is associated with talipes equinovarus (or ‘club foot’) and polydactyly.

Amelia

Amelia is a rare birth defect of lacking one or more limbs.

Genu valgus Genu valgus is commonly called ‘knocked knee’ and is a condition in which the knees angle in to touch each other and the distal limb is bent out laterally.

Genu varus Genu varus is commonly called ‘bowed legs’ and is a condition in which the knees bow outwards and the distal limb is bent in medially.

Meromelia Meromelia is a birth defect of lacking a part, but not all, of one or more limbs with the presence of a hand or foot. It results in a shrunken and deformed extremity.

Question 17 of 67 A 42-year-old woman felt a pop at the back of her leg while playing tennis and she presents with pain and weakness. On examination, she has a positive Thompson–Simmonds test. Which one of the following statements is true regarding the likely diagnosis?

Thompson’s test is lack of plantar flexion when calf is squeezed Thompson’s test is lack of dorsiflexion when calf is squeezed The condition affects women more than men The condition is most commonly due to direct impact trauma Plantar flexion is exaggerated by the condition

Explanation

Thompson’s test is lack of plantar flexion when calf is squeezed Thompson’s test is positive in Achilles’ tendon rupture, which is lack of plantar flexion when calf is squeezed.

Thompson’s test is lack of dorsiflexion when calf is squeezed Thompson’s test is lack of plantar flexion when calf is squeezed. The condition affects women more than men

Achilles’ tendon rupture is more common in men.

The condition is most commonly due to direct impact trauma Achilles’ tendon rupture is more commonly due to sudden dorsiflexion or plantar flexion of the ankle.

Plantar flexion is exaggerated by the condition

Dorsiflexion is exaggerated as it is normally limited by the Achilles’ tendon. Plantar flexion is limited.

Question 18 of 67

A patient develops a painful calf following surgery for a fractured tibia. They have increasing morphine requirements to remain comfortable. Which one of the following statements regarding compartment syndrome is true?

Pressure monitoring is required to confirm the diagnosis before surgical intervention Pain is aggravated by active stretch of the muscle group within the compartment If significant rhabdomyolysis occurs, this could result in hepatic failure Compartment syndrome is an emergency and immediate treatment is required Orthopaedic casts should remain on until the patient is taken to theatre to prevent infection

Explanation

Compartment syndrome is an emergency and immediate treatment is required Compartment syndrome should be treated as soon as possible with fasciotomies. Failure to treat leads to complications including tissue ischaemia and necrosis, Volkmann’s contracture, and rhabdomyolysis.

Pressure monitoring is required to confirm the diagnosis before surgical intervention Compartment syndrome is a clinical diagnosis. While pressure monitoring may confirm raised pressures, it is not required and can be misleading in certain cases.

Pain is aggravated by active stretch of the muscle group within the compartment Pain is made worse by passive stretching. Usually there will be a degree of pain on active stretching, however pain on passive stretch is a more concerning finding.

If significant rhabdomyolysis occurs, this could result in hepatic failure Rhabdomyolysis would cause renal failure, not hepatic failure.

Orthopaedic casts should remain on until the patient is taken to theatre to prevent infection Orthopaedic casts should be removed as soon as possible as this will reduce the intra-compartmental pressure instantly.

Question 19 of 67 You are assisting the paediatric orthopaedic consultant. They ask you about likely cases you will see in the clinic. What is the commonest pathological cause of the limping child aged 4–8 years?

Congenital dislocation of the hip (CDH) Juvenile Idiopathic Arthritis (JIA) Leukaemia Perthes' disease Slipped upper femoral epiphysis (SUFE)

Explanation

Perthes' disease Perthes’ disease is a condition in which avascular necrosis of the femoral head occurs. The incidence is approximately 1:10 000, the male to female ratio is 4:1. It presents between the ages of 4 and 8 years with a limp. The characteristic X-ray appearances may not become evident for up to 3 months after initial presentation.

Congenital dislocation of the hip (CDH) Congenital dislocation of the hip has an incidence of approximately 5–20 in 1000 live births, falling to roughly 1–2 per 1000 infants 3 weeks later due to spontaneous stabilisation. It is more common in girls. It usually presents in the first 3 years of life, although most cases are now diagnosed with neonatal screening.

Juvenile Idiopathic Arthritis (JIA) JIA is the classification for autoimmune arthritis in childhood replacing juvenile chronic/rheumatoid arthritis' and Still’s disease. Those affected are usually very young children, or older boys (aged 9 years and upwards).

Leukaemia Leukaemia should be considered in the differential diagnosis for ‘the limping child’, but it is not the most common pathological cause in this group.

Slipped upper femoral epiphysis (SUFE) Slipped upper femoral epiphysis presents between the ages of 10 and 16 years and is more common in boys than girls.

Question 20 of 67 A patient is referred to you from the obstetric baby checking service. They suspect a lower limb developmental abnormality. Which one of the following is correct regarding developmental dysplasia of the hip (DDH)?

Both hips are affected in the majority of patients In a newborn the femoral head will show on an ultrasound scan In the newborn the femoral head will show on a plain radiograph It affects about 1 in 10000 new borns It is equally common in boys and girls

Explanation

In a newborn the femoral head will show on an ultrasound scan Ultrasound scanning will show both the position of the femoral head and the shape of the acetabulum.

Both hips are affected in the majority of patients Both hips are affected in up to one-third of patients.

In the newborn the femoral head will show on a plain radiograph X-ray examination is of value in established cases, but in the newborn the femoral head is cartilaginous so it does not show on a plain radiograph.

It affects about 1 in 10000 new borns The incidence of developmental DDH is about 1 in 1000 newborns.

It is equally common in boys and girls

DDH is more common in girls.

Question 21 of 67

Hip surgery makes up a major part of orthopaedic emergency and elective work. Avascular necrosis of the femoral head is a severe complication. It is most commonly seen following which one of these injuries?

Anterior dislocation of the hip Displaced mid-shaft femoral fracture Intertrochanteric fracture of the femoral neck

Intracapsular fracture of the femoral neck Posterior dislocation of the hip

Explanation

Intracapsular fracture of the femoral neck Intracapsular neck of femur fractures are associated with disruption of the vascular supply to the femoral head. The incidence of avascular necrosis (AVN) is up to 15% in non-displaced fractures and increases to nearly 90% with untreated, completely displaced fractures.

Anterior dislocation of the hip

Anterior dislocation of the hip does not typically disrupt the vascular supply to the head of the femur and is not associated with avascular necrosis.

Displaced mid-shaft femoral fracture A displaced mid-shaft femoral fracture does not disrupt the blood supply to the femoral head.

Intertrochanteric fracture of the femoral neck Intracapsular, more than extracapsular, neck of femur fractures are associated with disruption of the vascular supply to the femoral head.

Posterior dislocation of the hip Posterior dislocation of the hip does not typically disrupt the vascular supply to the head of the femur and is not associated with avascular necrosis.

Question 22 of 67 Osteoarthritis and fractures in the hip are common. Arthroplasty is a definitive treatment From the following options, which one is the most common postoperative complication of total hip arthroplasty?

Asymptomatic Deep Vein Thrombosis Dislocation Heterotropic ossification Leg length discrepancy with Prosthesis infection

Explanation

Asymptomatic Deep Vein Thrombosis Thromboembolism occurs in 50% of cases of hip replacement, despite prophylaxis, but these rarely progress to fatal pulmonary embolism.

Dislocation Dislocation occurs in 2–5% of cases. Heterotropic ossification

Heterotopic ossification affects approximately 10% of cases.

Leg length discrepancy with Trendelenburg gait While this may occur after a total hip arthroplasty, thromboembolism is the more common postoperative complication.

Prosthesis infection Postoperative infection of the prosthetic joint is uncommon (<1%).

Question 23 of 67 Total hip arthroplasty is a routine procedure, however it can cause major complications. There are different techniques to complete the procedure, each with benefits and disadvantages. Which nerve is potentially at greatest danger of iatrogenic injury during total hip arthroplasty via the posterior approach?

Femoral nerve Lateral cutaneous nerve of the thigh Obturator nerve Sciatic nerve Superior gluteal nerve

Explanation

Sciatic nerve Sciatic nerve injury is the most common peripheral nerve injury occurring in total hip replacement.

Femoral nerve Iatrogenic injury to the femoral nerves is rare, accounting for less than 10% of all nerve injuries following hip surgery.

Lateral cutaneous nerve of the thigh Lateral skin incisions should be planned to avoid the lateral cutaneous nerve of the thigh.

Obturator nerve

Iatrogenic injury to the obturator nerves is rare, accounting for less than 10% of all nerve injuries following hip surgery.

Superior gluteal nerve

The superior gluteal nerve is retracted with the muscle intra-operatively and is unlikely to be injured.

Question 24 of 67

Pes cavus causes a foot deformity. In severe cases corrective surgery may be indicated. Which one of the following statements regarding pes cavus is true?

Pes cavus can be diagnosed using non-weight-bearing X-ray radiographs Compensatory utilisation of extensor digitorum longus to dorsiflex the ankle results in clawing of the toes Pes cavus is almost always asymptomatic and does not require treatment Most cases are idiopathic Pes cavus is a high arch of the foot that flattens with weight bearing

Explanation

Compensatory utilisation of extensor digitorum longus to dorsiflex the ankle results in clawing of the toes Clawing of the toes occurs when the intrinsic muscles develop contractures and the long extensor to the toes (extensor digitorum longus), is recruited to assist in ankle dorsiflexion. This causes claw toe deformity due to a change in its axis of pull.

Pes cavus can be diagnosed using non-weight-bearing X-ray radiographs Radiographs should be taken while weight bearing.

Pes cavus is almost always asymptomatic and does not require treatment Pes cavus may cause significant pain due to metatarsal compression.

Most cases are idiopathic The aetiology of pes cavus can be identified approximately 80% of the time. The remaining 20% of cases are idiopathic and non-progressive. The causes include malunion of calcaneal or talar fractures, burns, sequelae resulting from compartment syndrome, residual clubfoot, and neuromuscular disease. Identifying the aetiology is essential to determine if the deformity is progressive, which assists in operative planning.

Pes cavus is a high arch of the foot that flattens with weight bearing Pes cavus is a high arch of the foot that does not flatten with weight bearing.

Question 25 of 67

In pes planus, the whole foot is everted around its longitudinal axis and the medial arch comes into contact with the ground when standing. Which one of these anatomical structures is the dynamic stabilizer of the medial plantar arch?

Acetabulum pedis Calcaneo-navicular (spring) ligament Lisfranc joint Plantar fascia Tibialis posterior

Explanation

Tibialis posterior The tibialis posterior muscle and corresponding tendon are crucial to hindfoot position and foot flexibility during the gait cycle.

Acetabulum pedis The ‘acetabulum pedis’ maintains the medial longitudinal arch and acts as an important static stabilizer.

Calcaneo-navicular (spring) ligament The spring ligament complex is the most frequently affected static stabilizer in symptomatic pes planus.

Lisfranc joint The Lisfranc joint connects the tarsals and metatarsals. It does not have a role in stabilising the medial plantar arch.

Plantar fascia The plantar fascia is the aponeurosis that connects the heel to the toes. It does not have a role in stabilising the medial plantar arch.

Question 26 of 67

Dislocation of the hip joint in young adults is an orthopaedic emergency. Without prompt reduction this can result in a life changing injury. Which one of these complications increases in incidence the longer the hip remains dislocated?

Avascular necrosis of the femoral head Osteoarthritis Pulmonary embolism Recurrent hip dislocation

Sciatic nerve injury

Explanation

Avascular necrosis of the femoral head

Avascular necrosis of the femoral head occurs in 2–17% of patients. The longer it takes to relocate a hip, the higher the risk of avascular necrosis. Early relocation of a hip can make the difference between a healthy joint and a chronically disabled joint.

Osteoarthritis

Osteoarthritis is a recognised complication of hip joint dislocation, but its incidence is not related to delayed relocation.

Pulmonary embolism

Pulmonary embolism is a common complication for hospital patients – especially those who are postoperative or are poorly mobilising. There is no proven association, however, with delayed relocation following hip joint relocation.

Recurrent hip dislocation

There is no known association between delayed hip relocation and recurrent hip dislocation.

Sciatic nerve injury

Sciatic nerve injury is a recognised complication of hip joint dislocation, but its incidence has not been shown to increase with delayed relocation. Sciatic nerve injury may present as a foot drop.

Question 27 of 67 Osteochondritis of the navicular bone is known as Kohler’s disease. It presents with pain in the middle part of the foot. Which one of the following statements regarding Kohler’s disease is correct?

Avascular necrosis is the main pathological process that is thought to cause this condition Typically occurs in children aged 11–18 years old Ligamentous strain is the main pathological process that is thought to cause this condition Patients rarely become symptom free following treatment Children complain of pain on the lateral side of the foot.

Explanation

Avascular necrosis is the main pathological process that is thought to cause this condition It is thought to be due to a disturbance of the blood supply causing avascular necrosis and delay in ossification.

Typically occurs in children aged 11–18 years old Kohler’s disease typically occurs in children aged 3–5 years old.

Ligamentous strain is the main pathological process that is thought to cause this condition Avascular necrosis is the main pathological process that is thought to cause this condition.

Patients rarely become symptom free following treatment Normally symptoms disappear after a few weeks of strapping the foot and restricting activity, but rest in a cast may be necessary if there is severe pain. Eventually the foot becomes normal clinically and radiologically over a period of months.

Children complain of pain on the lateral side of the foot.

They complain of pain over the medial side of the foot and may present with a limp

Question 28 of 67 Developmental dysplasia of the hip where the acetabulum does not fully cover the femoral head. Left untreated it can cause early onset arthritis. Which one of the following is a test used to examine children with this condition and causes the hip to give a distinctive click when the hip is flexed and then abducted?

Barlow’s test Thomas’ test Kernig’s test Homan’s Ortolani’s test

Explanation

Ortolani’s test

Ortolani’s test causes the clunk of reduction as the hip is flexed and then abducted.

Barlow’s test

Barlow’s test causes the clunk of dislocation as the hip is adducted. Thomas’ test

Thomas’ test is used to exclude hip flexion contracture.

Kernig’s test Kernig’s test is used to test for meningism and is performed by flexing the hip to 90° and extending the knee.

Homan’s

Homan’s test, although not recommended generally, is a calf squeeze test to screen for deep vein thrombosis.

Question 29 of 67

Osteoporosis results in easy fractures. Recognition and treatment can prevent subsequent need for orthopaedic procedures. It is most reliably diagnosed by which one of the following techniques?

Dual-energy X-ray absorptiometry (DEXA) Measurement of serum osteocalcin Measurement of urinary collagen telopeptides Computed tomography (CT) scan Quantitative ultrasonography

Explanation

Dual-energy X-ray absorptiometry (DEXA) DEXA is currently the most precise and accurate method for diagnosing osteoporosis.

Measurement of serum osteocalcin Biochemical indices of bone formation (eg osteocalcin) are of value in monitoring the response of patients to treatment, but not for diagnosis.

Measurement of urinary collagen telopeptides Biochemical indices of bone breakdown (eg N-terminal and C-terminal crosslinking telopeptides of type-1 collagen) are of value in monitoring the response of patients to treatment, but not for diagnosis.

Computed tomography (CT) scan Quantitative CT scan of the lumbar spine is less precise (and more expensive, and involves a greater exposure to radiation).

Quantitative ultrasonography Quantitative ultrasound measurements on the calcaneum are being evaluated but have not been proven to be superior to DEXA.

Question 30 of 67

You notice an incidental bony lesion in a plain radiography. On showing the film to the orthopaedic consultant they recommend urgently bringing the patient to clinic. Which one of the following is the most common benign bone tumour affecting individuals under the age of 21 years?

Chondromyxoid fibroma Osteochondroma Giant-cell tumour Aneurysmal bone cyst Osteogenic sarcoma

Explanation

Osteochondroma An osteochondroma is a benign tumour that often protrudes from the outer contour of bones and is capped by growing cartilage. Around 80% of these lesions are noted before the age of 21 years.

Chondromyxoid fibroma Chondromyxoid fibroma is an extremely rare cartilaginous tumour.

Giant-cell tumour Giant-cell tumours are uncommon, mostly benign bone tumours that typically present in the fourth or fifth decade of life.

Aneurysmal bone cyst

Aneurysmal bone cysts are tumour-like lesions that typically present in childhood or adolescence, but are not as common as osteochondromas.

Osteogenic sarcoma Osteogenic sarcomas are not benign. They are aggressive malignant bone tumours of mesenchymal origin.

Question 31 of 67

A patient presents to the Orthopaedic Out-patient Clinic complaining of pain in his left tibia for the past 8 months. He said that the pain was particularly severe at night and was relieved by non-steroidal anti-inflammatory drugs. An X-ray is taken and is suggestive of an osteoid osteoma. Which one of the following is true regarding osteoid osteomas?

Osteoid osteomas are more common in women Osteoid osteomas typically present in the elderly The characteristic appearance on X-ray is a small radiopaque zone surrounded by a larger blastic zone Osteoid osteomas are typically benign tumours Osteoid osteoma is more common in the small bones of the body.

Explanation

Osteoid osteomas are typically benign tumours Osteoid osteomas are typically benign tumours arising from osteoblasts.

Osteoid osteomas are more common in women Osteoid osteomas are about three times more common in men.

Osteoid osteomas typically present in the elderly Osteoid osteomas tend to affect young adults.

The characteristic appearance on X-ray is a small radiopaque zone surrounded by a larger blastic zone The characteristic appearance on X-ray is a small radiolucent zone surrounded by a larger sclerotic zone.

Osteoid osteoma is more common in the small bones of the body. Osteoid osteoma is more common in the long bones.

Question 32 of 67 A patient presented to the Orthopaedic Out-patient Clinic complaining of pain in his right tibia, with a palpable swelling. X-ray of the affected bone is suggestive of a giant-cell tumour and revealed a lytic lesion involving the epiphysis and extending into the soft tissues. Which one of the following regarding giant-cell tumours is true?

Giant-cell tumours are typically benign Giant-cell tumours typically present in the elderly Giant-cell tumours typically occur in the diaphysis Giant-cell tumours appear osteoblastic on X-ray Giant-cell tumours rarely recur

Explanation

Giant-cell tumours are typically benign Rarely, a giant-cell tumour can metastasise, even though it remains histologically benign.

Giant-cell tumours typically present in the elderly Giant-cell tumours most commonly affect people in their twenties and thirties.

Giant-cell tumours typically occur in the diaphysis Giant-cell tumours typically occur in the epiphyses.

Giant-cell tumours appear osteoblastic on X-ray

Benign giant-cell tumours appear lytic on X-ray.

Giant-cell tumours rarely recur Giant-cell tumours are notorious for their tendency to recur.

Question 33 of 67 A 35-year-old woman typist in a busy legal firm notices that she develops tingling and numbness over the palmar surface of her thumb, index and middle fingers after several hours at her computer workstation. Pain in the same area often occurs at night as well. Which one of the following statements about the diagnosis is true?

The diagnosis is suspected further with a positive Tinel’s sign or Phalen’s sign

The causes of symptoms are due to compression of the ulnar nerve

Osteoporosis is often associated with carpal tunnel syndrome

The condition is most common in the elderly

The condition is more common in men

Explanation

The diagnosis is suspected further with a positive Tinel’s sign or Phalen’s sign Carpel tunnel syndrome is supported by Tinel’s sign (median nerve paraesthesia is reproduced by tapping at the volar surface of the wrist over the site of the median nerve in the carpal tunnel) and Phalen’s sign (reproduction of tingling with wrist flexion).

The causes of symptoms are due to compression of the ulnar nerve The symptoms of carpal tunnel syndrome are caused by compression of the median nerve in the carpal tunnel.

Osteoporosis is often associated with carpal tunnel syndrome Carpal tunnel syndrome is associated with rheumatoid arthritis. Other associations include diabetes mellitus, hypothyroidism, acromegaly, amyloidosis and pregnancy-induced oedema in the carpal tunnel.

The condition is most common in the elderly Carpal tunnel syndrome is most common in people aged 30–50 years.

The condition is more common in men Carpal tunnel syndrome is more common in women.

Question 34 of 67 A 72-year-old woman was treated non-operatively following an open fracture of the radial shaft. She returns to the hospital after having self-discharged despite the injury. Which one of the following early complications would you most likely expect to see?

Algodystrophy Infection Myositis ossificans Tendon rupture

Volkmann’s ischaemic contracture

Explanation

Infection Infection is an early complication of fractures, especially when open. Ideally, open fractures should be fixed within 4–6 h of injury. All patients with open fractures should be given antibiotics.

Algodystrophy Algodystrophy (Sudek’s atrophy) is characterised by a painful post-traumatic extremity. Late atrophic changes are seen, with features of patchy osteoporosis on X-ray.

Myositis ossificans Myositis ossificans is a form of heterotopic calcification occurring in soft tissue, notably after dislocation of the elbow or a blow to the brachialis, deltoid or quadriceps muscles. Treatment includes initial rest, then gentle active movements. Occasionally, excision of the bony mass is required.

Tendon rupture Tendon rupture is a well-recognised late complication of fractures. Extensor pollicis longus rupture occurs (6– 12 weeks) after fracture of the lower radius.

Volkmann’s ischaemic contracture Volkmann’s ischaemic contracture occurs following an arterial injury or compartment syndrome in which the muscle is replaced by inelastic fibrous tissue. This may lead to claw hand or toes.

Question 35 of 67 A 68-year-old man with known long-standing Paget’s disease presents with increasing pain, especially at night, in his left femur. He also complains of weight loss and night sweats. On clinical examination, there is a palpable mass over the distal third of the femur. What is the most likely diagnosis?

Abscess Ewing’s Sarcoma Fibrosarcoma Osteochondroma Osteosarcoma

Explanation

Osteosarcoma Osteosarcoma is a rare, but well-recognised, complication of Paget’s disease, and this accounts for the second peak in incidence around the sixth decade. It is most commonly found in the metaphysis of long bones and presents in the second or third decade of life in young adults, in the absence of Paget’s disease. Osteosarcomas may be osteolytic or osteoblastic or show a mixed picture on X-ray. Sun-ray spicules represent new bone formation once the cortex has been destroyed. Bone formation in Codman’s triangle is due to elevation of the periosteum by invasion of the tumour through the cortex.

Abscess The vignette is about a patient who is a known case of Paget’s disease and has developed a mass in the femur. The differential ranges from an abscess to malignancy. However, night sweats and weight loss are more suggestive of the development of malignancy, and it is osteosarcoma in such cases.

Ewing’s Sarcoma Development of a mass associated with night sweats and weight loss in a patient with a long-standing history of Paget’s disease indicates a malignancy. Ewing’s sarcoma is not known to develop in such patients.

Fibrosarcoma Features in the vignette are suggestive of the development of malignancy. Fibrosarcoma does not develop in long-standing cases of Paget’s disease.

Osteochondroma Osteochondroma does not develop in long-standing cases of Paget’s disease

Question 36 of 67

A 56-year-old woman has been diagnosed with an atrophic non-union in the mid-shaft of her left humerus. She complains of minimal discomfort at the fracture site, but she is unable to make best use of her arm, as there is movement at the fracture site. She feels depressed, as she would like to go back to work as a cashier at the local bank as soon as possible. She is otherwise fit and well. What is the most appropriate line of treatment?

Electrical stimulation Intramedullary nailing Bracing for 6 weeks Plating and bone grafting Vitamin D and calcium supplements

Explanation

Plating and bone grafting The highest union rate (98%) is observed in osteoporotic patients where autologous bone graft is used in combination with plating.

Electrical stimulation The cause of non-union in this post-menopausal woman is most probably osteoporosis. Electrical stimulation is less likely to induce osteoblastic activity and new formation, and hence union rates are not high. This patient requires autologous bone graft with some sort of bone fixation.

Intramedullary nailing In osteoporotic bone, the union rate is less for intramedullary nailing alone (66%), when compared with autologous bone graft combined with plating (98%).

Bracing for 6 weeks Bracing for 6 weeks will not induce new bone formation. The best management with the highest success rate is plating with autologous bone graft.

Vitamin D and calcium supplements The reason for non-union is most probably osteoporosis. Prescription of vitamin D and calcium supplements is less likely to induce new bone formation. Non-union is caused by: unrecognised delayed union; wide distraction and separation of the fracture surfaces; soft tissue interposition; excessive movement at the fracture site; and poor local blood supply. Painless movement at the fracture site is diagnostic of non-union, as distinct from delayed union which is painful. Two types of non-union are described: hypertrophic (bulbous bone ends) and atrophic (no calcification at bone ends).

Question 37 of 67 A 64-year-old gentleman presents with a hard lump and pain in his proximal femur. He also complains of increasing headaches and deafness. Radiographs of the femur show new bone formation and sclerosis which causes thickening of the cortex and coarse trabeculae. The laboratory results reveal a marked increase in alkaline phosphatase levels, and the histology results demonstrate excessive osteoclast resorptive activity and paratrabecular fibrosis. Which of the following is the first-line agent for treatment of this patient’s condition?

Bisphosponates Calcitonin Calcium and Vitamin D Cortisol Parathyroid hormone

Explanation

Bisphosponates Bisphosphonates suppress the osteoclastic activity. These can help regulate bone growth and reduce pain. Examples include risedronate, zoledronate and pamidronate. The most common side-effect of risedronate is gastritis, while that of pamidronate is flu-like symptoms. History, examination and radiological findings are suggestive of Paget’s disease of bone. It is characterised by enlargement and thickening of the bone, but the internal architecture of the bone is abnormal and the bone is unusually brittle. It affects men and women equally, most commonly from the age of 50 years. The most commonly affected sites are the pelvis and tibia. An elevated alkaline phosphatase level indicates increased osteoblastic activity. Complications include nerve compression, fractures, osteoarthritis, osteosarcoma, high- output cardiac failure and hypercalcaemia. Treatment focuses on suppressing bone turnover and includes the use of bisphosphonates as a first-line agent and calcitonin. Surgery may be required for pathological fractures, osteoarthritis and nerve entrapment.

Calcitonin Calcitonin is a second-line treatment for suppression of osteoclastic activity. It is prescribed in patients who do not tolerate bisphosphonates. Calcitonin is given as a daily injection.

Calcium and Vitamin D Calcium and vitamin D alone do not relieve the symptoms of Paget’s disease. These are given to all the patients, especially those on bisphosphonates.

Cortisol Cortisol may have some role in the management of cardiac failure in patients with Paget’s disease; however, it is not a first-line treatment.

Parathyroid hormone Parathyroid hormone causes bone resorption, hence it will worsen the symptoms. Calcitonin is a treatment of Paget’s disease which blocks the effects of parathyroid hormone

Question 38 of 67 A patient suffers an ankle injury. On plain radiography an avulsion fracture is present. Which one of the following mechanisms is associated with avulsion fractures?

Powerful muscle contractions Axial loading Direct blow Cortical bone destruction Twisting

Explanation

Powerful muscle contractions Avulsion fractures are caused when the muscle or tendon contracts vigorously, so avulsing the piece of bone at the insertion of the bone. This type of fracture is common in the knee and shoulder joints.

Axial loading Axial loading causes compression fractures. It is common in falling from a height with landing on the feet. This can lead to crush fractures of the spine. The common findings are compression and/or wedging of the vertebral bodies.

Direct blow Depressed fractures suggest a direct blow or shearing force. The forces tend to be localised. For example, a blow from a hammer to the skull can cause a depressed skull fracture. Comminuted fractures are usually caused due to a high-velocity injury. In this type of fracture, the bone is shattered into pieces, consisting of small and large fragments.

Cortical bone destruction Metastasis to the long bones can lead to cortical bone destruction. Even a trivial force can cause a pathological fracture. Occasionally, pathological bone fractures are the first presentation of an underlying malignancy. Bone biopsy in such cases may help in the diagnosis of an occult malignancy.

Twisting Twisting can cause spiral fractures. A typical example is when the foot is caught during playing football and the player rotates the leg on a fixed foot. It leads to a spiral fracture of the tibia. Because of the flexible nature of the bones, a greenstick fracture is more common in children. This type of fracture does not extend to both cortices of the bone. The healing in this type of fracture is usually quick and efficient. In compound fractures, the fractured bone protrudes through the skin, or a soft tissue wound leads to the fracture site. The risk for infection is highest in compound fractures.

Question 39 of 67 A 29-year-old man, who is an active smoker, is seen in the Combined Orthoplastics Clinic 4 months after having had a tibial nail and free anterolateral thigh (ALT) flap for an open tibial fracture. There is no evidence of callus formation at the fracture site on plain radiography. However, the surgeons plan for no additional intervention. Which of the following is most likely the cause of delayed union?

Micro-movement at the fracture site Smoking Myocutaneous flap cover Internal fixation over external fixation Use of gentamicin beads at time of debridement

Explanation

Smoking Smoking can impair fracture union, wound healing and the success of a free flap. It is likely that smoking exerts these effects through impairing blood flow to healing tissues.

Micro-movement at the fracture site Although too much movement at a fracture site can delay healing, ‘micro-movements’ promote callus formation.

Myocutaneous flap cover Myocutaneous flaps have better outcomes than fasciocutaneous flaps. However, success of flaps depends on the integrity of the blood supply. Cigarette smoking decreases the chances of success.

Internal fixation over external fixation In Gustilo type IIIB open tibial fractures, in the absence of infection, better outcomes are seen with internal fixation and ALT free flaps. This approach reduces the risk for complications and the inconvenience of external fixators.

Use of gentamicin beads at time of debridement Gentamicin beads are used to prevent infection and increase the outcome of fracture. Non-steroidal anti- inflammatory drugs (NSAIDs), however, can cause impairment of blood supply to the graft and increased chances of non-union/delayed union. Delayed union is the term applied when a fracture has not healed within the period of time that would be considered adequate for bone healing for that particular site. It is expected that union will eventually occur without further intervention. For tibial fractures, healing should occur in 16 weeks. Non-union is the term applied to a fracture that will not unite without additional intervention. The time limit is usually 6–9 months for tibial fractures. The causes of a delayed union of fracture include inadequate blood supply, infection (including acute and chronic osteomyelitis), excessive shearing movements between the fragments, loss of apposition between the fragments, over-distraction of the bony fragments by excessive traction, dissolution of the fracture haematoma by synovial fluid (may occur in fractures within joints), the presence of a foreign body in the fracture site and the presence of a tumour in the fracture site. Question 40 of 67

A 29-year-old man undergoes intramedulary nailing for a closed fracture of his left tibia. During the night, you are asked to see him as he complains of pain and his usual medications are not settling him. On examination, his left leg is swollen and tense. Distal pulses are not palpable. Which one of the following is the most likely finding on dipstick urinalysis?

Haemoglobinuria Alkalotic urine Red blood cell (RBC) casts Glucosuria High urinary creatine phosphokinase (CPK)

Explanation

Haemoglobinuria The mechanism of trauma, the surgical procedure and the findings on clinical examination are suggestive of compartment syndrome. Ischaemic injury to muscle leads to rhabdomyolysis. Myoglobin is released in the urine and detected as haemoglobin on the urine dipstick – the dipstick would be false positive for haemoglobin. Compartment syndrome is typically due to trauma, eg haemorrhage after fracture, surgery or blunt trauma or from post-ischaemic swelling. It can occur after any injury, including crush injuries, following prolonged compression of a limb, such as a tight cast, and prolonged surgery. It occurs more commonly in the leg and forearm. The most important symptom is pain out of proportion to the injury. Unless an arterial injury has occurred, peripheral pulses remain normal in most cases of acute compartment syndrome. The 4 Ps are pain, paraesthesiae, paralysis and pulselessness; however, these appear late and it is recommended not to wait till these signs appear. Management is emergency fasciotomy.

Alkalotic urine Urine is acidic in compartment syndrome due to associated metabolic acidosis. Alkalinisation of urine is a therapeutic measure to prevent renal damage. It makes myoglobin soluble in the urine and help in excretion.

Red blood cell (RBC) casts RBC casts are a feature of glomerulonephritis, and not of compartment syndrome. Glucosuria

There is no cause for glucose to be found in the urine. The patient is not noted to be diabetic.

High urinary creatine phosphokinase (CPK) CPK is a measure in serum, and not in urine. Increased levels of serum CPK in the range of 1000– 5000 units/ml or greater suggest compartment syndrome. Combined with the presence of myoglobinuria, it is diagnostic of compartment syndrome. The appearance of CPK may suggest impending renal failure.

Question 41 of 67 A 60-year-old woman presents with a painful knee. The pain has been gradually increasing, is worse at the end of the day and with exercise and is associated with a swollen joint. Her full blood count (FBC), erythrocyte sedimentation rate (ESR) and C-reactive protein (CRP) are all normal. Which of the following is the most likely condition?

Septic arthritis Osteoarthritis Rheumatoid arthritis (RA) Gout Reactive arthritis.

Explanation

Osteoarthritis Osteoarthritis presents with a long-standing history. The pain is worse with exercise and occurs towards the end of the day. As it is not an inflammatory condition, so it does not cause any such blood result abnormalities. The knee is very commonly affected by osteoarthritis.

Septic arthritis Septic arthritis is inflammation of a joint caused by a bacterial infection. Septic arthritis has a more acute presentation. Any joint can be affected by septic arthritis, but it is most common in the knees and hips. Symptoms include pain and tenderness over a joint, pain on moving the joint and feeling unwell. It is an uncommon, but serious, infection. Urgent treatment is needed. This includes antibiotic medicines and drainage of the infected fluid from the joint to prevent permanent joint damage.

Rheumatoid arthritis (RA) RA is a condition that causes inflammation in many joints of the body, which particularly affects the hands, feet, wrists, ankles and knees. It tends to occur symmetrically. The pain is relieved by exercise and is worse on waking/resting. RA causes anaemia of chronic disease and, during flare-ups, raised ESR and CRP levels.

Gout Gout is a form of inflammatory arthritis characterised by recurrent attacks of a red, tender, hot and swollen joint. Pain typically comes on rapidly in < 12 hours. The joint at the base of the big toe is most commonly affected. Diagnosis is by finding high levels of uric acid in blood and negatively birefringent crystals on synovial fluid examination. Treatment of an acute attack is by painkillers. Prevention of attacks depends upon the underlying cause of gout, ie increased production vs decreased excretion.

Reactive arthritis. Reactive arthritis is a chronic form of arthritis that often occurs following an infection of the genital, urinary or gastrointestinal system. It may involve the knee joint. However, there is no association with physical activity.

Question 42 of 67

A 16-year-old girl presents with a 4-month history of an aching discomfort in the distal right femur, relieved with simple analgesia. A plain radiograph of the knee shows an area of slight sclerosis. Which is the most likely diagnosis?

Chondroma Fibroma Osteochondroma Osteoclastoma Osteoid osteoma

Explanation

Osteoid osteoma Osteoid osteomas are most common in the femur and tibia. They are unusual as benign tumours, in that they produce a constant aching pain which is classically relieved by simple analgesia. They can be difficult to see on plain radiographs.

Chondroma Chondromas are made up almost entirely of cartilage and are common in the hands and feet. Radiological appearance is seen as a saucerisation of the adjacent bony cortex, with a sclerotic periosteal reaction.

Fibroma A non-ossifying fibroma (also called fibroxanthoma) is a common benign bone tumour in children and adolescents. These are well-circumscribed lytic lesions. No treatment is needed in asymptomatic patients, and spontaneous remission with replacement by bone tissue may happen.

Osteochondroma Osseocartilaginous exostoses are the most common benign tumours of the bones. These often appear as a bony pedicle growing away from the epiphyseal plate, covered in a large cartilage cap. Classic radiographic features include orientation of the lesion away from the physis and medullary continuity.

Osteoclastoma Osteoclastomas are commonly found lying close to the epiphyseal plate, with destruction of the overlying cortex. These are also called giant cell tumours. Common sites are around the knee joint. A typical radiological finding is a soap bubble appearance

Question 43 of 67 A 75-year-old lady has presented with a fracture due to osteoporosis. She suffers from many medical conditions and currently is on a variety of medicines. Which one of the following is the most likely cause of osteoporosis in this patient?

Hormone replacement therapy (HRT) Hypoparathyroidism

Thyrotoxicosis

Osteoarthritis

Warfarin Explanation

Thyrotoxicosis Thyrotoxicosis causes acceleration of bone remodelling, and hence secondary osteoporosis. The extent of reduction in bone density is variable; however, it is reversible.

Hormone replacement therapy (HRT) HRT reduces the risk for osteoporosis.

Hypoparathyroidism Hyperparathyroidism, and not hypoparathyroidism, causes secondary osteoporosis.

Osteoarthritis Osteoporosis and osteoarthritis are two very different medical conditions. There is little in common. These conditions develop differently, have different symptoms, are diagnosed differently and are treated differently. No relationship could be found between the two. If anything, the relationship is inverse.

Warfarin Although initially suspected, long-term warfarin use has not been found to increase the risk for osteoporosis. Causes of osteoporosis are primary – related to the ageing process and decreased gonadal activity (most common). Secondary causes can be subdivided into:

 nutritional: scurvy, malabsorption, malnutrition  endocrine: hyperparathyroidism, thyrotoxicosis, Cushing’s disease, gonadal insufficiency  drug-related: corticosteroids, alcohol, heparin  malignant disease: carcinomatosis, leukaemia, multiple myeloma  systemic disease: rheumatoid arthritis (RA), tuberculosis (TB), chronic liver disease (especially primary biliary cirrhosis), ankylosing spondylitis  idiopathic: juvenile osteoporosis, post-climacteric osteoporosis.

Weight-bearing exercise and oestrogen, whether endogenous or as HRT, reduce the risk for osteoporosis.

Question 44 of 67

A 19-year-old sportsman complains of pain in his lower thigh, especially at night. It is not relieved with simple analgesia. On clinical examination there is a palpable mass over the distal third of the femur. X-ray reveals Codman’s triangle. Which one of the following is the most likely diagnosis?

Osteochondroma Osteoblastoma Osteoid osteoma Ewing’s sarcoma Osteosarcoma

Explanation

Osteosarcoma Codman’s triangle is most likely to be a characteristic feature of osteosarcoma. Osteosarcoma has two peaks of incidence: 10–25 years and over 65 years, the second peak being due to its development in those with Paget’s disease. One per cent of patients suffering from Paget’s disease of bone develop a high-grade osteosarcoma. The clinical features of osteosarcoma include pain, local tenderness, and soft tissue mass. Pathological fractures are rare. The tumour tends to spread towards the medulla and towards the periosteum, resulting in the X-ray appearances which may be variable. The characteristic changes include the ‘sunburst’ effect due to new bone radiating outwards from the breached cortex, and Codman’s triangle due to new bone forming at the angles of periosteal elevation secondary to tumour emerging from the cortex. The common sites include the metaphysis of the femur (52%), proximal end of tibia (20%) and the humerus (9%). The male to female ratio is 3:2. The lung is a common site for metastatic disease. While localised disease has a survival rate of 70 to 75%, metastasis to the lungs or other bones is associated with a survival rate of about 30%. The mainstay of management is adequate surgical excision and chemotherapy. Limb-sparing surgery is possible in 80% of cases. Prognosis is better in young adults and those with more distally located tumours. Multifocal osteosarcomas and those with a background of Paget’s disease have a poor prognosis.

Osteochondroma Osteochondroma is the most common benign tumour of the bone. Due to the overlying cartilaginous cap, it is called an osteochondroma. X-rays reveal the osteochondroma as a projection of the bone. Prognosis is good as the lesion stops growing once growth is complete, however very rarely malignant transformation may occur.

Osteoblastoma Osteoblastoma is a benign tumour of the bone originating from precursor cells. The most common site is within the vertebrae. X-rays reveal a core of sclerotic bone surrounded by a hypodense ring. Pain is not relieved by aspirin, which helps in differentiating this from an osteoid osteoma.

Osteoid osteoma This is a benign tumour of the bone originating from osteoblasts. The metaphysis of the femur and the tibia are commonly affected. Pain is characteristically relieved by aspirin. X-rays reveal a core of sclerotic bone surrounded by hypodense ring.

Ewing’s sarcoma Ewing’s sarcoma is a malignant tumour of bone originating from neuroectodermal cells. It spreads from the medullary cavity to the cortex and tends to involve the diaphysis of long bones. Male children are affected more than women. X-rays reveal an onion skin sign. Histopathology shows small blue round undifferentiated cells that suggest an origin of this tumour from neuroectodermal cells. Treatment involves excision, chemotherapy and radiotherapy. Codman's can be a feature of Ewing's sarcoma, but it is not the most likely of the options given.

Question 45 of 67

An 18-month-old child presents with features suggestive of septic arthritis of the left hip. They are taken to theatre for a washout of the affected joint. Which one of the following is the most likely causative organism?

Neisseria gonorrhoeae Haemophilus influenzae Staphylococcus aureus Chlamydia trachomatis Salmonella

Explanation

Staphylococcus aureus

Staphylococcus aureus infection is the most common cause of acute bacterial arthritis in children and adults. The most common predisposing condition to this infection in adults is prosthetic joints.

Neisseria gonorrhoeae

Neisseria gonorrhoeae is the most common organism among younger sexually active individuals, therefore it is not the most common cause in children less than 2 years of age.

Haemophilus influenzae

Haemophilus influenza used to be a common cause of septic arthritis in children less than 2 years of age, however due to introduction of vaccination, its incidence has decreased.

Chlamydia trachomatis

Reactive arthritis is seen commonly in patients with gastrointestinal (GI) or genitourinary infections. The most common pathogens are of GI origin however Chlamydia trachomatis genitourinary infection is the second most common cause of reactive arthritis. Reactive arthritis is immunological in origin and is not septic.

Salmonella

This organism is a more common cause of septic arthritis and osteomyelitis in sickle-cell disease patients.

Question 46 of 67

Which one of the following orthopaedic procedures is an absolute indication for prophylactic antibiotics?

Carpal tunnel decompression Diagnostic arthroscopy Diagnostic aspiration of a joint for infection Intra-articular joint injection Total hip replacement

Explanation

Total hip replacement Prophylactic antibiotics are indicated in any joint replacement and whenever a prosthesis/foreign body is implanted

Carpal tunnel decompression Antibiotics are not required for carpal tunnel decompression as the procedure is grouped as clean and it does not involve insertion of any implant or prosthetic material.

Diagnostic arthroscopy Diagnostic arthroscopy does not require antibiotic prophylaxis. It is a clean procedure.

Diagnostic aspiration of a joint for infection Antibiotics are contraindicated before joint aspiration for infection as they could give false-negative results.

Intra-articular joint injection Antibiotics are not usually required for joint injection unless the patient is immunocompromised.

Question 47 of 67 A 46-year-old farmer sustained a penetrating injury to his index finger. Two days later, the digit is swollen, and painful to move. On examination the index finger is held in slight flexion, passive extension of the digit is painful and there is tenderness throughout the flexor sheath of the index finger Which one of the following organism is most likely responsible for this condition?

Herpes virus Candida albicans Eikenella corrodens Pasteurella multocida Staphylococcus aureus

Explanation

Staphylococcus aureus

The most probable diagnosis is bacterial flexor tenosynovitis. It is usually caused by Staphylococcus aureus following a penetrating injury. Bacterial flexor tenosynovitis should be distinguished from stenosing tenosynovitis (‘trigger’ finger) of the digital flexor tendons. Stenosing tenosynovitis mostly affects the thumb or ring finger but can involve any digit. Bacterial flexor tenosynovitis, on the other hand, results from penetrating wounds to the hand and the majority are due to Staphylococcus aureus infection. Any digit can be affected depending on the site of injury. Infections of the finger flexor tendons are serious, with the potential for substantial permanent functional impairment. Finger movements are severely restricted and the four cardinal signs of tendon sheath infection are: partially flexed posture of the finger; fusiform swelling of the finger; tenderness along the entire flexor tendon sheath; and severe pain from passive extension of the finger. The tenosynovitis can rapidly destroy the synovial gliding surfaces and result in healing with restrictive adhesions. A fulminating infection can lead to tendon necrosis. Early tenosynovitis can be treated with intravenous antibiotics, elevation and splinting. However, if no definite signs of improvement are seen after 24 h, surgical exploration or drainage must be considered as the consequences of a fulminating flexor tenosynovitis are so severe and permanent. Initial drainage should be with a limited proximal incision in the distal palm over the tendon sheath, through which a small catheter can be introduced. A small distal incision for outflow is then made to create a through-and-through irrigation system with minimal surgical damage. For the vast majority of cases these measures, along with antibiotics and supportive care will bring the situation promptly under control. In severe cases, however, radical drainage by opening the whole finger through a mid-axial incision and opening the sheath while carefully preserving the pulleys may be required.

Herpes virus Clinical findings are suggestive of pyogenic tenosynovitis. Herpes virus causes herpetic whitlow infection of the hand, usually on the fingers. Healthcare workers are prone to develop this as their hands are exposed to the saliva of patients carrying herpes virus. Findings include small, swollen, painful blood-tinged blisters, and sometimes numbness. The condition usually resolves in several weeks without any treatment or morbidity.

Candida albicans

Candida albicans causes chronic paronychia. It is more likely to occur with chronic immersion in water (as in dishwashers), previous trauma, or nail defects. It can be treated with topical antifungal agents. Occasionally, excision of a portion of the nail or removal of the entire nail may be necessary. Eikenella corrodens

Eikenella corrodens is a fastidious facultative anaerobic Gram-negative bacillus that is present as endogenous microbiota in the mouth and upper respiratory tract as well as on other mucosal surfaces. It is associated with hand infections caused by human or animal bites.

Pasteurella multocida

Pasteurella multocida is a small, Gram-negative bacteria. It is responsible for hand infections caused by animal bite, scratch, or lick.

Question 48 of 67

A 33-year-old intravenous (IV) drug user (IVDU) is admitted to the emergency department with septic shock and rapidly progressing cellulitis of his right upper limb. After admission infection has spread to the abdominal wall over a period of few hours. Which one of the following is most important step in management of this patient?

Antibiotic therapy Wide excision of the skin and necrotic fascia Hyperbaric oxygen therapy Conservative wound debridement Femoral angioplasty

Explanation

Wide excision of the skin and necrotic fascia Management of necrotising fasciitis requires liberal debridement of the skin and underlying fascia. Skin loss will require plastic procedures and negative wound therapy to cover the skin loss following debridement. In the early stages of necrotising fasciitis the skin appears normal; if untreated subcutaneous oedema develops with gangrene of the dermal layer. The infection is typically polymicrobial eg streptococci, bacteroides, haemolytic streptococci, often developing from a simple inoculation, or occurring in a post-operative wound. It requires aggressive treatment, with resuscitation, high-dose antibiotics, and radical debridement of the fascial planes affected. Antibiotic therapy alone is insufficient to limit the spread of necrotising fasciitis due to its aggressive nature and the definitive treatment remains early and aggressive surgical intervention. Delay often results in increased morbidity and mortality. If the perineum is involved, a colostomy is beneficial to help maintain wound cleanliness. The mortality is in the order of 30% and has remained static despite advances in intensive care. This is likely to be due to the difficult nature of diagnosing necrotising fasciitis early.

Antibiotic therapy This patient is suffering from necrotising fasciitis. Antibiotic therapy is part of treatment. However, definitive treatment requires liberal debridement.

Hyperbaric oxygen therapy There is no role of hyperbaric oxygen therapy in management of necrotising fasciitis.

Conservative wound debridement Liberal debridement is required. Infection is usually beyond the level of skin discoloration. The underlying fascia should be debrided until it starts bleeding.

Femoral angioplasty

Clinical presentation is not suggestive of acute thrombo/embolic event of the limb. Hence there is no role of angioplasty. Question 49 of 67 A 56-year-old woman with known insulin-dependent diabetes and obesity presents with erythema and swelling of the face. She rapidly becomes unwell. Examination reveals crepitus in the subcutaneous tissue. Contrast computed tomography (CT) reveals air pockets in the underlying muscles. Which one of the followings is the most likely pathogen?

Streptococcus pneumoniae Clostridium perfringens Mycobacterium tuberculosis Pseudomonas Staphylococcus aureus

Explanation

Clostridium perfringens

The presence of the air pockets in the underlying muscles is suggestive of gas gangrene. Air pockets limited to subcutaneous tissues suggest necrotising fasciitis. The organism involved in gas gangrene is Clostridium perfringens. It may occur after trauma or surgery when it is wound related, In certain immunocompromised patients C. perfringens can causes spontaneous gas gangrene. These include patients with colorectal malignancy, diabetics and patient with colitis. Treatment includes antibiotics, wound debridement and hyperbaric oxygen.

Streptococcus pneumoniae

It causes respiratory infection in diabetics.

Mycobacterium tuberculosis

It causes pulmonary or extra pulmonary tuberculosis.

Pseudomonas

Pseudomonas is responsible for malignant or necrotising otitis externa in diabetic patients older than 35 years. It also causes diabetic foot infections. However, the description in the vignette is suggestive of gas gangrene for which the organism is C. perfringens.

Staphylococcus aureus

Staphylococcus aureus causes diabetic foot infections and necrotising fasciitis. The involvement of muscles makes this organism unlikely.

Question 50 of 67 A 7-year-old boy is referred to an orthopaedic clinic with pain in his left forearm that has been getting progressively worse. His only medical history of note is a fracture of his left distal radius requiring manipulation and K-wire fixation 6 months earlier. X-rays of the left distal radius shows changes consistent with osteomyelitis. Which one of the following organisms is most likely to be responsible?

Gonococcus

Meningococcus

Pneumococcus

Staphylococcus

Salmonella Typhi

Explanation

Staphylococcus

Staphylococcus aureus infection is the most common cause of acute bacterial arthritis in children and adults. The most common predisposing condition to this infection in adults is prosthetic joints. Presence of a K-wire at fracture site makes it vulnerable to infection by Staphylococcus aureus. Staphylococcus aureus infection is the most common cause of acute bacterial arthritis in children and adults. The most common predisposing condition to this infection in adults is prosthetic joints. Positive blood cultures are only found in about 50% of proven cases of infection. Staphylococci are the most common causative organisms, but Streptococcus, gonococcus and pneumococcus are also responsible. Haematogenous spread may occur from a skin abrasion, a tooth abscess, a boil, any indwelling line or catheter, and in newborns an infected umbilical cord.

Gonococcus Gonococcus (Neisseria gonorrhoeae) causes sexually transmitted disease (STD)-related arthritis.

Meningococcus Meningococcus (Neisseria meningitidis) does not cause any bone infections.

Pneumococcus Bone infection caused by a pneumococcus (Streptococcus pneumoniae) is extremely rare. It accounts for less than 2% of acute osteomyelitis in children.

Salmonella Typhi

This is the common cause of septic arthritis and osteomyelitis in sickle-cell disease patients

Question 51 of 67 A 60-year-old man has presented with discharge from his right foot. Examination reveals a red swollen foot with weak pulses and impaired sensation. The patient has had diabetes for last 15 years. Which one of the following investigations is of highest diagnostic value?

Ankle–brachial pressure index Bone scan Magnetic resonance imaging (MRI) scan Radiographs Transcutaneous oxygen pressures

Explanation

Magnetic resonance imaging (MRI) scan Magnetic resonance imaging is the most sensitive and most specific imaging modality for the detection of osteomyelitis and provides superb anatomic detail and more accurate information of the extent of the infectious process and soft tissues involved. The diagnostic imaging of osteomyelitis can require the combination of diverse imaging techniques for an accurate diagnosis. Conventional radiography should always be the first imaging modality to start, as it provides an overview of the anatomy and the pathological conditions of the bone and soft tissues of the region of interest. Ultrasonography is most useful in the diagnosis of fluid collections, periosteal involvement, and surrounding soft tissue abnormalities and may provide guidance for diagnostic or therapeutic aspiration, drainage, or tissue biopsy. A computed tomography scan can be a useful method to detect early osseous erosion and to document the presence of sequestrum, foreign body, or gas formation but generally is less sensitive than other modalities for the detection of bone infection. Nuclear medicine imaging is particularly useful in identifying multifocal osseous involvement.

Ankle–brachial pressure index It may suggest impaired arterial blood flow, however it has limited value in the diagnosis of underlying osteomyelitis.

Bone scan

The role of bone scans is controversial in the diagnosis of osteomyelitis as the numbers of false- positive and false-negative results are high.

Radiographs

Plain radiographic studies of the diabetic foot may demonstrate demineralisation and Charcot joint and occasionally may suggest the presence of osteomyelitis. However it has high false-negative rate as it may fail in picking up osteomyelitis presenting at an early stage.

Transcutaneous oxygen pressures Transcutaneous tissue oxygen studies are done in cases of vaso-occlusive disease. The indications include borderline situations in which the advisability of arterial bypass surgery may be unclear. It has no role in investigation of osteomyelitis. Transcutaneous oxygen pressures (TcpO2) are considered to be the gold standard to assess wound healing potential. Question 52 of 67

A 27-year-old male patient has sustained a single minimally displaced fracture of the tibial shaft. Their case is being discussed at the morning trauma meeting. Which one of the following is a risk factor for delayed union?

A closed fracture A high consumption of dairy products An intact fellow bone Pain at the fracture site Treatment in an above-knee cast

Explanation

An intact fellow bone Distraction of fragments causes delayed fracture healing. Application of traction, or intact fellow bone (in this case the fibula) causes distraction of bone fragments and hence delayed union.

A closed fracture This is favourable for early healing.

A high consumption of dairy products This helps in bone healing.

Pain at the fracture site Pain is not directly related to non/delayed union, however, there are some controversial reports about non-steroidal anti-inflammatory drugs (NSAIDS) and opioids. Studies have reported that fracture healing may be delayed due to the anti-inflammatory effects of analgesics.

Treatment in an above-knee cast Minimal manipulation of the fracture haematoma is protective. Hence when indicated an above-knee cast is protective. In contrast, internal fixation can disturb the fracture haematoma and can cause delayed union. Causes of delayed union include an inadequate blood supply, infection, incorrect immobilisation and an intact fellow bone (such as the fibula in a tibial fracture). The fracture site is usually tender and the fracture remains visible, with very little callus formation, periosteal reaction or sclerosis at the bone-ends. Continued treatment is required and includes functional bracing, an excellent method of promoting bony union. A fracture should undergo internal fixation and bone grafting if union is delayed for more than 6 months and in the absence of any callus formation.

Question 53 of 67

A 25-year-old man presents with pain in the lower back. Examination reveals loss of lumbar lordosis, and restriction of spinal movements. He also has a positive Schober’s test and reduced chest expansion. Of the following options, what is the most likely diagnosis based on this presentation?

Ankylosing spondylitis

Cauda equina syndrome

Lumbar disc herniation

Spinal stenosis

Spondylolisthesis

Explanation

Ankylosing spondylitis

Loss of lumbar lordosis with restriction of spinal movements in a young patient are highly suggestive of ankylosing spondylitis. Ankylosing spondylitis is a systemic chronic autoimmune disease, causing sacroiliitis, progressive spinal deformities and hip osteoarthritis. Patients present with stiffness. Radiographs show the classical bamboo spine appearance. Treatment is symptomatic, with physiotherapy. Spinal decompression and fusion is needed if there is a progressive neurological deficit.

Cauda equina syndrome

The presentation of cauda equina syndrome is with sudden onset of pain in the lower limbs, associated with loss of control on bowel and bladder function. It is a surgical emergency and requires emergent decompression. The causes include any condition that causes narrowing of the spinal canal and compresses lower nerve roots.

Lumbar disc herniation

Most disc herniations occur either at the L4–L5 or L5–S1 discs. Presentation may vary depending upon the level and severity of nerve root compression. Loss of lumbar lordosis may occur due to spasm and contraction of paravertebral muscles, however movements of the spine are not compromised. A magnetic resonance injury (MRI) scan is diagnostic. Treatment may be conservative or surgical depending upon severity of disease.

Spinal stenosis

Any narrowing of the spinal canal can lead to spinal stenosis. It may be a developmental abnormality or a degenerative process, symptoms can mimic cauda equina syndrome. Degenerative spinal stenosis is a disease of the elderly. Restriction of spinal movement is not a feature.

Spondylolisthesis The abnormal forward slip of one vertebral body on another is called spondylolisthesis. Severe cases can cause a narrowing of the spinal canal and lead to cauda equina syndrome. Loss of movements of the spine is not a feature.

Question 54 of 67

A 25-year-old man is admitted with a glass injury to the wrist. Examination reveals an inability to flex the middle finger at the distal interphalangeal joint (DIP). Which one of the following structures is most likely to be divided?

Ulnar nerve Median nerve Tendon of the flexor digitorum profundus (FDP) to the middle finger Tendon of flexor digitorum superficialis (FDS) to the middle finger Flexor carpi radialis x-axis

Explanation

Tendon of the flexor digitorum profundus (FDP) to the middle finger Tendons of FDP insert into the distal phalanges of the four fingers and flexes the distal interphalangeal joint.

Ulnar nerve Injury of ulnar nerve at wrist will lead to paralysis of all the muscles supplied by ulnar nerve in hand.

Median nerve Injury of the median nerve at the wrist will lead to paralysis of all the muscles supplied by the median nerve in hand, it also well covered at the wrist and unlikely to be injured as it passes through the carpal tunnel.

Tendon of flexor digitorum superficialis (FDS) to the middle finger Tendons of FDS insert into middle phalanx of the four fingers. It flexes proximal interphalangeal joints and assists in flexing the metacarpophalangeal joints. The tendon of FDP is the only flexor tendon that inserts into the distal phalanx and flexes the distal interphalangeal joint.

Flexor carpi radialis x-axis Flexor carpi radialis inserts into the base of the second and third metacarpal bones. It flexes and abducts hand at wrist joint. It is not involved in the movement of the digits.

Question 55 of 67

A 54-year-old woman presents with burning pain associated with tingling and numbness in right hand. She wakes up at night due to pain. Examination reveals altered sensation on the palmar aspect of the right hand in the area of thumb, index and middle fingers, and radial half of the ring finger. Compression of which nerve is responsible for his symptoms?

Axillary nerve Median nerve

Musculocutaneous nerve

Radial nerve

Ulnar nerve

Explanation

Median nerve The median nerve is the only nerve that passes through the carpal tunnel. It supplies sensation to palmar surface of thumb and radial two and a half fingers including nails. Compression in carpal tunnel leads to altered sensation in the area of distribution. It spares thenar eminence as sensory branch from median nerve to this area passes above the flexor retinaculum. In advanced cases muscles supplied by median nerve in hand also become weak.

Axillary nerve The diagnosis of the condition described is carpel tunnel syndrome. The axillary nerve does not supply hand and does not pass below the flexor retinaculum.

Musculocutaneous nerve The musculocutaneous nerve does not supply the hand.

Radial nerve The diagnosis of the condition described is carpel tunnel syndrome. The radial nerve supplies the thumb and radial one and half fingers on the dorsum surface of the hand, excluding nails. Moreover it does not pass below the flexor carpal retinaculum.

Ulnar nerve The ulnar nerve passes outside of the flexor retinaculum. In the hand it supplies the skin over the medial one and half fingers both on the palmar and dorsal aspects

Question 56 of 67

A 47-year-old man is referred to the plastic surgery team as he has sustained a deep wound to his hand while using a circular saw at home. The bleeding has been controlled with pressure, but he has altered sensation in the median nerve distribution and it is thought he has a flexor tendon injury as he is unable to flex his middle and ring fingers. On examination the wound is 5 cm long and situated medial to the thenar eminence – starting at the wrist crease. What zone of tendon injury would be present in this position?

Zone I

Zone II

Zone III

Zone IV

Zone V Explanation

Zone IV Zone IV injuries include tendon injuries occurring in the area overlying the carpal tunnel. Bleeding and involvement of median nerve suggests injury to this area. Although neurovascular structures are involved both in zones IV and V injuries. However in the description the injury lies distal to wrist crease which is an area of zone IV. Zone V injuries lie proximal to wrist crease.

Zone I Zone I injuries include tendon injuries occurring in the area between the distal interphalangeal (DIP) and proximal interphalangeal (PIP) joint creases. Zone I injuries involve the FDP tendons only. The median nerve is not involved in this area.

Zone II Zone II injuries include tendon injuries occurring in the area between the midpoint of the middle phalanx and distal palmar crease. Zone II must be carefully treated as there is a close relationship with the FDS and the FDP, which can cause adhesion formation and failed repair. The median nerve is not involved in this area.

Zone III Zone III injuries include tendon injuries occurring in the area between the distal palmar crease and the distal margin of the carpal tunnel. The median nerve is not involved in this area.

Zone V Zone V injuries include tendon injuries occurring in the area between the forearm and wrist up to the proximal border of the carpal tunnel. Bleeding and involvement of median nerve suggests injury to this area. Although neurovascular structures are involved both in zones IV and V injuries. However in the description the injury lies distal to wrist crease which is an area of zone IV Zone V injuries lie proximal to wrist crease. Management of tendon injuries depend on the area where tendons are involved. The zones for flexor and extensor injuries are different. For a flexor tendon injury the zones as described by Verden, are:  Zone I – Between the distal interphalangeal (DIP) and posterior interphalangeal (PIP) joint creases.  Zone II – Between the midpoint of the middle phalanx and distal palmar crease.  Zone III – Between the distal palmar crease and the distal margin of the carpal tunnel.  Zone IV – Overlying the carpal tunnel.  Zone V – The forearm and wrist up to the proximal border of the carpal tunnel.

This patient also has signs of a median nerve injury which is vulnerable in this location and another important reason to formally explore the wound in theatre. Question 57 of 67

A 10-year-old child is brought in by his mother with a three-day history of sudden onset hip pain and inability to weight bear. He is pyrexial and feels unwell. Blood tests show an elevated white-cell count and a raised C-reactive protein level. What is the most likely diagnosis?

Hip dysplasia

Irritable hip

Perthes disease

Reactive arthritis

Septic arthritis Explanation

Septic arthritis

Septic arthritis presents with an acute onset of pain and inability to weight bear. The child lies with the affected leg in flexion and external rotation to reduce pain. The most common causative organism is Staphylococcus aureus. Diagnosis could be confirmed with joint aspiration. Treatment is with urgent wash-out of the hip joint and intravenous antibiotics. Complications include femoral head destruction and osteonecrosis.

Hip dysplasia Hip dysplasia is a non-infective condition and presentation is not like the one described in the vignette. Developmental dysplasia of the hip (DDH) is termed congenital hip dislocation or hip dysplasia. Screening is done at birth to detect this condition. Without early diagnosis the outcome is poor.

Irritable hip Irritable hip is a non-infective condition and the presentation is not like the one described in vignette. It is also termed transient synovitis. It is the most common cause of a limp in children. It presents with an acute onset of hip pain and a limp that gradually resolves.

Perthes disease

Perthes’ disease is a non-infective condition and presentation is not like the one described in the vignette. Perthes’ disease is also known as Legg–Calvé–Perthes’ disease or avascular necrosis of head of femur. It is a self-limiting disease of the femoral head comprising of necrosis, collapse, repair, and remodelling.

Reactive arthritis

Reactive arthritis is a non-infective condition and the presentation is not like the one described in the vignette. It is also known as Reiter’s syndrome. It develops in response to urinary or gastrointestinal tract infection. It may take a few months to resolve

Question 58 of 67

A 78-year-old woman presents with pain in both hands and is dropping. Examination reveals that her middle finger proximal interphalangeal joint is fixed in flexion of approximately 30° and the distal interphalangeal (DIP) joint is fixed in hyperextension. How would you describe this finding?

Bouchard’s node Boutonnière deformity Heberden’s nodes Mallet finger Swan neck deformity

Explanation

Boutonnière deformity The deformity described in the vignette is characteristic of a boutonnière deformity. Boutonnière is the French word for ‘buttonhole’.

Bouchard’s node

Bouchard’s nodes are seen in osteoarthritis. Although found at proximal interphalangeal joints (PIP), these are hard, bony outgrowths and not a fixed flexion/ extension deformity.

Heberden’s nodes

Heberden’s nodes are seen in osteoarthritis. These are hard, bony outgrowths similar to Bouchard’s nodes but are found on the distal interphalangeal joints.

Mallet finger

Mallet finger occurs due to injury to the extensor mechanism to the distal phalanx. It is also known as baseball finger, however any force that causes hyperextension at the DIP can lead to a mallet finger. The resultant deformity is fixed flexion at DIP.

Swan neck deformity

Both the posterior interphalangeal and DIP are involved in a swan neck deformity. Characteristically, there is hyperextension at the posterior interphalangeal and fixed flexion at DIP. Looking at finger gives the impression of swan neck. This deformity is the opposite to that found in boutonnière’s deformity.

Question 59 of 67

An 8-year-old boy presents to the emergency department with pain in the left hip and a limp. The symptoms have been present for 24 h, but the boy has had previous episodes of hip pain intermittently for the past 3 months. He is small for his age, systemically well, does not recall any injury and his bloods are normal. What is the most likely diagnosis?

Developmental dysplasia of the hip

Perthes disease Septic arthritis Slipped upper femoral epiphysis Transient synovitis

Explanation

Perthes disease Perthes’ disease is a transient disruption in the blood supply to the femoral head, which is more common in boys, those who are small for their age and between the ages of 3–12 years. Onset is over weeks to months and pain is intermittent. The hip is tender on and flexion and internal rotation are commonly affected.

Developmental dysplasia of the hip Developmental dysplasia of the hip is most commonly diagnosed in babies, and is screened for, but may present late usually when the child starts walking.

Septic arthritis Septic arthritis can occur at any age, but is more common in younger babies, it is generally associated with systemic upset and raised inflammatory markers.

Slipped upper femoral epiphysis Slipped upper femoral epiphysis is also more common in boys, but tends to occur in older children, most commonly 11–16 years, at the time of increased pubertal growth.

Transient synovitis Irritable hip (transient synovitis) is a non-infective condition. It is the most common cause of a limp in children. Onset is acute and over a few days. Pain occurs at the end of the arc of motion. The child is otherwise well. However, transient synovitis is a diagnosis of exclusion. The condition usually resolves without recurrence/ consequences.

Question 60 of 67

A 50-year-old man presents with pain in his right shoulder. The pain started 2 days ago, after he had painted a ceiling. The pain is present on forward flexion and abduction between 70–120°. Hawkins’ test is positive, Jobe’s test, internal and external rotation lag signs, and Gerber’s lift-off tests are negative. What is the most likely diagnosis?

Acromioclavicular (AC) joint arthritis Acute calcific tendonitis

Glenohumeral joint arthritis

Rotator cuff tear Subacromial impingement

Explanation

Subacromial impingement Subacromial impingement is tested by Hawkins–Kennedy impingement sign. During this test, the patient’s shoulder is placed into 90° of forward flexion; the elbow is placed in 90° flexion and then passively internally rotated. Presence of pain suggests that space between the humeral head and acromion process has been reduced. Internal rotation causes the supraspinatus tendon or the subacromial bursa to become compressed and elicit a painful response.

Acromioclavicular (AC) joint arthritis AC joint arthritis present with pain over the superior aspect of the shoulder. Pain is often worse with internal rotation which is tested by asking the patient to place the arm behind the back. Radiography and magnetic resonance imaging (MRI) scan of the shoulder may be required to make diagnosis.

Acute calcific tendonitis Acute calcific tendonitis tends to present with sudden, acute, severe shoulder pain, with pain present even at rest and on all movements.

Glenohumeral joint arthritis Osteoarthritis of the main shoulder joint cavity presents with deep joint pain that is exacerbated by extensive use, often described as an ache. is reduced and examination may reveal stiffness and crepitus. X-rays and other imaging studies are helpful in diagnosis.

Rotator cuff tear The rotator cuff consists of four muscles (supraspinatus, infraspinatus, teres minor and subscapularis). Individual muscles may be tested for possible tear. Jobe’s test is positive for the supraspinatus muscle; the internal rotation lag sign and Gerber’s lift-off test are positive for the subscapularis muscle; and the external rotation lag test is positive in case of infraspinatus muscle involvement. Absence of these clinical signs make rotator cuff tears less likely.

Question 61 of 67

Examination of a newborn reveals a ‘clunk’ when a slight gentle posterior pressure is applied to the hips in adducted position. The midwife is concerned about developmental dysplasia of the hip and asks the orthopaedic team to review the child. What is the most appropriate next step in the management of this baby?

Immediate ultrasound (US) scan of the hips Ultrasound (US) scan of the hips at 4 weeks Ultrasound (US) scan of hips at 14 weeks X-ray of hips at 2 weeks X-ray of hips at 4 weeks

Explanation

Ultrasound (US) scan of the hips at 4 weeks If in a newborn reveals abnormalities, then a US scan is requested. The timing of the scan is at 2 weeks, to reduce splint age in children that do not require it, as the majority of lax capsules will tighten up by this stage.

Immediate ultrasound (US) scan of the hips Examination findings suggest developmental dysplasia of the hip (DDH). Test described in the vignette is called Barlow’s test. US on urgent basis may lead to high false-positive rates due to lax capsules. The optimum time is 4-6 weeks. Within the first 2–3 weeks most babies’ hips stabilise spontaneously without the need for treatment.

Ultrasound (US) scan of hips at 14 weeks Delaying the US scan by 14 weeks will lead to delay in diagnosis and management (application of harness/ brace).

X-ray of hips at 2 weeks The femoral capital epiphysis does not begin to ossify until 3–4 months of age, therefore X-rays are only helpful in diagnosis if presentation is delayed.

X-ray of hips at 4 weeks

X-rays are only helpful in diagnosis if presentation is delayed.

Question 62 of 67

A 26-year-old man sustained a transverse displaced tibia fracture following a football injury. The fracture was manipulated in theatre and placed in an above-knee back slab. Which symptom is the earliest sign of a developing compartment syndrome of the leg?

Decreased pulses in the foot Pain on active flexion of toes Pain out of proportion to injury to the clinical situation Palpable swelling Paralysis

Explanation

Pain out of proportion to injury to the clinical situation The most common early symptom of impending compartment syndrome is pain out of proportion to injury.

Decreased pulses in the foot

Presence of distal pulses are not reassuring. One should not wait till disappearance of pulses as it is a late sign and irreversible damage may have occurred by this time.

Pain on active flexion of toes Pain on passive stretching is more suggestive of compartment syndrome. Pain on active movement of toes may be present even in the absence of compartment syndrome due to fracture of the bone.

Palpable swelling Swelling is common after injuries and may occur in the absence of compartment syndrome. However, a tense tender leg should raise suspicion of compartment syndrome.

Paralysis

Pulselessness, paralysis, pallor, and paraesthesia are all late symptoms and signs and should not be waited for to make the diagnosis of compartment syndrome

Question 63 of 67

A 45-year-old man sustains a displaced intracapsular fracture of his neck of femur following a motor bike accident. You are consenting him for theatre and he has some concerns about what is being proposed. What is the most likely complication of this injury?

Avascular necrosis Femoral artery thrombosis

Non-union Post-traumatic osteoarthritis

Sciatic nerve palsy

Explanation

Avascular necrosis

Femoral neck fractures are high-energy injuries in young patients. Avascular necrosis (AVN) is the most common complication after femoral neck fractures due to the disruption of blood supply. The risk of AVN in a displaced intracapsular neck of the femur fracture is approximately 30–40%. Risk increases with delayed surgical fixation and therefore emergent operative treatment is recommended in the young patient to enable fixation and avoid arthroplasty.

Femoral artery thrombosis

Injury to femoral vessels in trochanteric fractures is rare. The profunda femoris artery is most commonly involved. The mechanisms include pressure of a sharp bone fragment (the lesser trochanter), the tip of protruding cortical screws (extramedullary implant) or the distal locking screw (gamma nail).

Non-union

Conservative treatment of a displaced fracture can lead to a painful non-union, hence operative management is recommended. A recent metanalysis has shown a non-union (NU) rate of about 9% in intracapsular hip fractures in young adults.

Post-traumatic osteoarthritis

Secondary osteoarthritis is very rare in patients with hip fractures (less than 0.5%).

Sciatic nerve palsy Sciatic nerve injury is rare. It may be associated with hip replacement and displacement of the implant. It leads to foot drop and carries significant lower limb morbidity. Its incidence in total hip replacement varies from zero to 2.8%. Question 64 of 67

A 75-year-old woman tripped and fell at home. She was brought to the emergency department with a painful hip. On examination her leg was shortened and externally rotated. X-rays demonstrated an intertrochanteric proximal femoral fracture. Which method of treatment is best for this injury?

Cannulated screws fixation

Hip hemiarthroplasty Hip spica cast Internal fixation with Dynamic Hip Screw Skin traction

Explanation

Internal fixation with Dynamic Hip Screw Internal fixation with dynamic hip screw is associated with highest chances of fracture healing. It provides a chance for early mobilisation and weight bearing. The blood supply to the femoral head comes through the capsule. Therefore extracapsular fractures do not usually disturb the vascularisation. This situation means that extracapsular fractures can be fixed with a high union rate. A dynamic hip screw is the most commonly used device for intertrochanteric neck of femur fractures.

Cannulated screws fixation Cannulated screw fixation is used for intracapsular neck of femur fractures.

Hip hemiarthroplasty Intracapsular fracture are treated with hip hemiarthroplasty.

Hip spica cast It is not a preferred treatment due to the high failure rate and immobility-associated complications.

Skin traction Non-operative treatment of neck of femur fractures are associated with very high complications rate due to immobility, and not recommended except in patients who are not fit for anaesthetics.

Question 65 of 67

An otherwise fit and well 25-year-old patient fractured his right tibia and is being treated non-operatively in an above-knee plaster. You see him in the clinic after his admission and the check X-ray demonstrates good progress. What is the minimum time interval required for callus to become visible on radiographs?

1 week 2–3 weeks 4–6 weeks 6–8 weeks 1–2 years

Explanation 2–3 weeks Callus is formed and fluffy opacity appears on X-ray. This phase is labelled as phase of callus formation, which involves formation of scaffolding for new bone.

1 week After fracture, the inflammatory process starts rapidly and lasts until fibrous tissue, cartilage, or bone formation begins (1–7 days post fracture). Initially, there is haematoma formation and inflammatory exudation from ruptured blood vessels. Visibility of callus requires the formation of scaffold that occurs during the second to third weeks.

4–6 weeks Union phase occurs during this period. A bridge of cartilage or immature bone is formed between the two ends. At this stage the fracture is stable but still weak.

6–8 weeks This is a phase of consolidation. Callus is replaced by bone and immature bone is replaced by lamellar bone.

1–2 years This is the stage of remodelling. It is the final stage. Continued osteoblast and osteoclast activity causes reshaping of the fracture site. As a result the fracture site achieves best bone density and shape.

Question 66 of 67

An 87-year-old woman is referred to an orthopaedic clinic with ongoing left hip pain that has persisted for the last year following a fractured hip. This was treated with cannulated screws and an X-ray shows changes consistent with avascular necrosis of the femoral head. Which one of the following statements regarding avascular necrosis is correct?

Fractures of the scaphoid bone may be complicated by avascular necrosis The neck of femur is particularly susceptible to avascular necrosis Bisphosphonates protect against avascular necrosis Females are more commonly affected Symptoms develop quickly after the process of avascular necrosis begins

Explanation

Fractures of the scaphoid bone may be complicated by avascular necrosis A scaphoid fracture often severs the proximal fragment from the blood supply entering the distal aspect of the bone.

The neck of femur is particularly susceptible to avascular necrosis The head (not the neck) of the femur undergoes necrosis when the sub-retinacular arteries are severed following fractures of the neck.

Bisphosphonates protect against avascular necrosis Bisphosphonates are associated with avascular necrosis of the jaw.

Females are more commonly affected Males are more commonly affected

Symptoms develop quickly after the process of avascular necrosis begins

Patients are often asymptomatic at the beginning of avascular necrosis

Question 67 of 67

A patient presents with an open fracture, they have travelled from abroad back to the UK and therefore operation is delayed. Post-operatively they are diagnosed with osteomyelitis. Which one of the following statements regarding osteomyelitis is correct?

The long bones are the least likely to be affected by osteomyelitis Salmonella infection is the most common cause of osteomyelitis in the general population Ultrasound is typically the first line imaging investigation of osteomyelitis Staphylococcus aureus is the most common causative species in the general population Trauma is a rare cause of osteomyelitis

Explanation

Staphylococcus aureus is the most common causative species in the general population In both adults and children, Staphylococcus aureus is the most common causative species that causes osteomyelitis. Note that a patient may develop osteomyelitis without an open fracture.

The long bones are the least likely to be affected by osteomyelitis The long bones (of the upper and lower limbs) are the most likely to be affected by osteomyelitis.

Salmonella infection is the most common cause of osteomyelitis in the general population Salmonella is more commonly found as a causative species in patients with sickle-cell anaemia

Ultrasound is typically the first line imaging investigation of osteomyelitis Plain radiographs are more commonly used as a first-line imaging investigation in osteomyelitis. Ultrasound can be used to assess soft tissue changes, but cannot image within the bone.

Trauma is a rare cause of osteomyelitis Trauma can account for up to 47% of osteomyelitis cases.